2sat Fire

You might also like

Download as pdf or txt
Download as pdf or txt
You are on page 1of 128

SAT Firebrick

SAT
Firebrick

Reading / Writing
Reading / Writing

Name
SAT Reading Index

SAT Reading Test 1 2p


SAT Reading Test 2 24p
Test 1 Reader’s Guide 41p
Test 2 Reader’s Guide 51p

SAT Writing Index

Writing Concept & Practice Questions 62p


1. Number Agreement
2. Pronoun Errors
3. Parallelism Errors
4. Comparison Errors
5. Modifier Errors
6. Punctuation
7. Structure

Writing Practice Test 1 95p

Writing Practice Test 2 111p


SAT Reading

11
SAT
Reading Test 1

22
3
Q u e sti
o ns 1-10 are b ased o n the fo l
l
o w i
ng see that the ring was not continuous. Its broken
p ass ag e. circle was made of sharply edged cubes about an
3
0

T hi
s passage i
s adapte
d f
rom T he M et
al M onste
r, by A . inch in height, separated from each other with
M err i
tt
. mathematical exactness by another inch of space. I
counted them—there were nineteen.
We passed through what had once been a great Almost touching them with their bases were an
door into another chamber larger than that we had just equal number of pyramids as sharply angled and of
3
5
left; and it was in better preservation, the ceiling similar length. They lay on their sides with tips
L
i
ne unbroken, the light dim after the blazing sun of the pointing starlike to six spheres clustered like a
5 court. Near its center she halted us. conventionalized five petaled primrose in the exact
Before me ran a two-feet-wide ragged crack, center. Five of these spheres—the petals—were, I
splitting the floor and dropping down into black roughly calculated, about an inch and a half in
4
0
depths. Beyond was an expanse of smooth flagging, diameter, the ball they enclosed larger by almost an
almost clear of debris. inch.
10 Drake gave a low whistle. I followed his pointing So orderly was their arrangement, so much like a
finger. In the wall at the end whirled two enormous geometrical design nicely done by some clever child
dragon shapes, cut in low relief. Their gigantic wings, that I hesitated to disturb it. I bent, and stiffened, the
4
5
their monstrous coils, covered the nearly unbroken first touch of dread upon me.
surface, and these CHIMERAE were the shapes upon For within the ring, close to the clustering globes,
15 the upthrust blocks of the haunted roadway. was a miniature replica of the giant track in the
In Ruth's gaze I read a nameless fear, a half poppied valley.
shuddering fascination. 5
0 I reached down and picked up one of the
But she was not looking at the cavern dragons. pyramids. It seemed to cling to the rock; it was with
Her gaze was fixed upon what at my first glance effort that I wrenched it away. It gave to the touch a
20 seemed to be a raised and patterned circle in the slight sensation of warmth—how can I describe it?—a
dust-covered floor. Not more than a foot in width, it warmth that was living.
shone wanly with a pale, metallic bluish luster, as 5
5 I weighed it in my hand. It was oddly heavy,
though, I thought, it had been recently polished. twice the weight, I should say, of platinum. I drew
Compared with the wall's tremendous winged figures out a glass and examined it. Decidedly the pyramid
25 this floor design was trivial, ludicrously insignificant. was metallic, but of finest, almost silken texture—and
What could there be about it to stamp that dread upon I could not place it among any of the known metals.
Ruth’s face?
I leaped the crevice; Dick joined me. Now I could

U n
a
u
t
h
o
r
i
z
e
d
c
o
p
y
i
n
g
o
r
r
e
u
s
e
o
f
a
n
y
p
a
r
t
o
f
t
h
i
s
p
a
g
e
i
s
i
l
l
e
g
a
l
. 2

4
25
It certainly was none I had ever seen; yet it was as
60
1
certainly metal. It was striated—slender filaments radiating
from tiny, dully lustrous points within the polished surface. Which choice best summarizes the passage?
And suddenly I had the weird feeling that each of these A) Adventurers undergo transformation that causes
points was an eye, peering up at me, scrutinizing me. permanent loss.
65There came a startled cry from Dick. B) Adventurers enter a place where they face a
“Look at the ring!” threatening creature.
The ring was in motion! C) Adventurers cooperate to fight against a
Faster the cubes moved; faster the circle revolved; the mysterious enemy.
pyramids raised themselves, stood bolt upright on their D) Adventurers discover a form of precious metal.
square bases; the six rolling spheres touched them, joined
70
the spinning, and with sleight-of-hand suddenness the ring
drew together; its units coalesced, cubes and pyramids and
globes threading with a curious suggestion of ferment.
With the same startling abruptness there stood erect, 2
where but a moment before they had seethed, a little figure,
75
In line 3, “preservation” most nearly means
grotesque; a weirdly humorous, a vaguely terrifying A) security
foot-high shape, squared and angled and pointed and B) shape
ANIMATE—as though a child should build from nursery
C) storage
blocks a fantastic shape which abruptly is filled with
D) control
throbbing life.
80
A troll from the kindergarten! A kobold of the toys!
Only for a second it stood, then began swiftly to
change, melting with quicksilver quickness from one outline
into another as square and triangle and spheres changed
places. Their shiftings were like the transformations one sees
85
within a kaleidoscope. And in each vanishing form was the
suggestion of unfamiliar harmonies, of a subtle, a 3
transcendental geometric art as though each swift shaping
The description of Ruth's reaction “What...face?”
were a symbol, a WORD—
90 Euclid's problems given volition! (lines 26-27) mainly serves to
Geometry endowed with consciousness! A) draw a comparison
It ceased. Then the cubes drew one upon the other until B) intensify the suspense
they formed a pedestal nine inches high; up this pillar C) illustrate a personality
rolled the larger globe, balanced itself upon the top; the D) explain an event
five spheres followed it, clustered like a ring just below it.
95
The other cubes raced up, clicked two by two on the outer
arc of each of the five balls; at the ends of these twin
blocks a pyramid took its place, tipping each with a point.
The Lilliputian fantasy was now a pedestal of
1
0cubes surmounted by a ring of globes from which
0
sprang a star of five arms.
The spheres began to revolve. Faster and faster
they spun around the base of the crowning globe; the
arms became a disc upon which tiny brilliant sparks
105appeared, clustered, vanished only to reappear in
greater number.
A troll swept toward me. It GLIDED. The finger
of panic touched me. I sprang aside, and swift as
light it followed, seemed to poise itself to leap.

U n
a
u
t
h
o
r
i
z
e
d
c
o
p
y
i
n
g
o
r
r
e
u
s
e
o
f
a
n
y
p
a
r
t
o
f
t
h
i
s
p
a
g
e
i
s
i
l
l
e
g
a
l
. 3

265
4 6
The passage most clearly implies that Ruth is In line 57, “decidedly” most nearly means
terrified because she witnesses A) emphatically
A) a distorted geometric pattern of the ring B) determinedly
B) the massive size of the pyramids C) definitely
C) the sharpness of the surrounding rocks D) exceedingly
D) the path inside the discovered ring

7
The narrator indicates that the appearance of "a little
figure" (line 75) is
A) absurd and sudden.
B) small and pleasant.
C) threatening and powerful.
D) uncanny and aggressive.
5
Which choice provides the best evidence for the
answer to the previous question?
A) Lines 34-36 ("Almost...length")
B) Lines 43-45 ("So...it")
C) Lines 47-49 ("For...valley!")
D) Lines 50-51 ("I...away")

3 3

U n
a
u
t
h
o
r
i
z
e
d
c
o
p
y
i
n
g
o
r
r
e
u
s
e
o
f
a
n
y
p
a
r
t
o
f
t
h
i
s
p
a
g
e
i
s
i
l
l
e
g
a
l
. 4

627
8 10
The narrator implies that Drake gives a low whistle Which choice provides the best evidence for the
because answer to the previous question?
A) he wants to send a signal to his friends. A) Lines 99-101 ("The...arms")
B) he tries to alleviate Ruth's horror. B) Lines 102-103 ("Faster...globe")
C) he is impressed by the scenery that he C) Lines 103-106 ("the...number")
encounters. D) Lines 108-109 ("I...leap")
D) he can sing the tune that the figures react to.

9
As presented in the passage, the troll is best
described as
A) moving in a circular motion.
B) constantly growing in size.
C) possessing a phenomenal speed.
D) having a complex structure.

U n
a
u
t
h
o
r
i
z
e
d
c
o
p
y
i
n
g
o
r
r
e
u
s
e
o
f
a
n
y
p
a
r
t
o
f
t
h
i
s
p
a
g
e
i
s
i
l
l
e
g
a
l
. 5

287
Q u e sti
on s 11 -
20 ar e base d o n th e fol
lo w i
n g departures and still get drivers to their destinations
passage and supplementary materials faster than otherwise. Of course such a ledger of
4itineraries is highly unrealistic, but Google search
5
T his passage isad a
p t
e d f
rom " P red ic
ting Road Condit
ions
data might be a good proxy.
wt h
i Int
er n
e tS e
arch", byNk o
ilao s A ski
tas.
What an individual searches for in Google reveals
Adverse road conditions such as traffic congestion something about their state or their intent. If they
are known to cause a host of undesired side effects. search for a recipe, we know what they might cook;
These vary from increased pollution, energy waste, 5if they search for the bus departure timetable, they
0
additional transportation and production costs to waste might take the bus; and if they search for
of labor and delays in product deliveries. Traffic
L
i
n
e information on traffic congestion along a certain
congestion
5 is also known to impact public health highway, they are likely to drive on it at a later
negatively, to cause stress and road rage and to even point in time. The core observation which sparked
affect the unborn. If we thought of a city, a region, a 5this method is that every morning searches in
5
country or any other social unit as a large living Germany peak at 7 a.m., and then they start to
organism, road traffic would be one of its circadian dissipate as drivers are being injected into the traffic.
rhythms and traffic jams would be an obstruction to
1
0 Two hours later, at 9 a.m., we have the morning
its entrainment. It is hence not surprising that peak of ADAC traffic reports. Similarly at 4 p.m.
obstructing the smooth flow of traffic sends ripples of every afternoon, we observe the search peak. Two
60
negative effects deep into many aspects of hours later, at 6 p.m., we see a peak of the ADAC
socioeconomic life. Understanding, forecasting and traffic reports. Clearly, performing a Google search
preventing
1
5 adverse road conditions is therefore and driving at the same time are mutually exclusive
important for the benefit of the drivers but also and this is what creates this advance.
obviously for economic reasons. 65 A future proliferation of mobile applications which
The emergence of traffic jams is a complex matter allow one to speak searches into a cell phone as well
and may have many causes. Some are simple and as to graphically get current crowdsourced traffic
have to do with fluctuating degradation of the
2
0 conditions may diminish the effectiveness of this
available infrastructure (e.g. accidents, road method. But currently, such search intensity is a good
constructions, weather and the like) and some are proxy for the number of people who intend to drive
70
more complex and depend on behavioral, topological soon. By searching for traffic jam information ahead
and dynamical complexities. There is an extensive of driving, drivers provide us with advance warning
literature
2
5 both empirical and theoretical which develops of their intent to drive. To my knowledge there is no
“fundamental diagrams of traffic” i.e. studies the system available which can have as much as two
relationships between traffic density, velocity and flow. hours advance on the emergence of such road
75
The basic intuition in this type of research is that the conditions. It does so not by using an algorithm
higher the traffic density is the more likely it trained on past values of road conditions but by
becomes
3
0 that driver actions are interlinked: high using behavioral measurements guaranteed to occur
density implies more or less that if one driver breaks before the emergence of road congestion. Such
then all drivers do so as well with an increasing current “floating car data” systems can predict at
80
abruptness. Once a critical density is reached traffic most 15-30 minutes ahead.
will almost certainly stall. Forecasting and preventing
traffic jams is hence a hard problem.
3
5
If we had a global ledger where future traffic
participants would volunteer their itineraries in
advance, things might be different. We could then get
an advance estimate of future traffic and even try to
prevent traffic density from reaching critical mass by
4
0
deploying an algorithm by which we queue

U n
a
u
t
h
o
r
i
z
e
d
c
o
p
y
i
n
g
o
r
r
e
u
s
e
o
f
a
n
y
p
a
r
t
o
f
t
h
i
s
p
a
g
e
i
s
i
l
l
e
g
a
l
. 6

829
Figure 1 11
What function does the third paragraph (lines 37-46)
serve in the passage as a whole?
A) It lists different current theories regarding the
various causes of traffic problems.
B) It acknowledges that solving traffic problems
requires technologies that are currently out of
reach.
C) It introduces an instrument that can be utilized
to devise a solution to the problem presented
earlier.
D) It provides a historical overview of the traffic
problems and an outlook on their solutions.
F
igm Go
u
r
e
a
d
o
p
t
e
d
f
r
o o
g
l
e
A n
a
l
y
t
i
c
s
,
2
0
1
8.

12
Figure 2
Which choice does the author explicitly cite as a
difficulty in forecasting traffic jams?
A) A single driver's action can influence a large
number of other drivers.
B) Google searches do not yet provide the
necessary means for predicting traffic jams.
C) Accidents and road constructions happen too
frequently to estimate traffic jams accurately.
D) Complexity of factors that create traffic jams is
beyond human comprehension.

13
Which choice provides the best evidence for the
Figure adopted from Pr
e
dic
t
i
ngR
oad
Con
d
i
t
i
o
n
sanswer to the previous question?
wt
h
i
I
nternetSear
ch by Nko
il
ao
s
A s
k
it
as
,2
01
6 A) Lines 20-25 ("Some...complexities")
B) Lines 25-28 ("There is...flow")
C) Lines 31-34 ("high...abruptness")
D) Lines 34-35 ("Once...stall")

U n
a
u
t
h
o
r
i
z
e
d
c
o
p
y
i
n
g
o
r
r
e
u
s
e
o
f
a
n
y
p
a
r
t
o
f
t
h
i
s
p
a
g
e
i
s
i
l
l
e
g
a
l
. 7

9
30
14 16
The central idea of the fourth paragraph (lines As used in line 38, "itineraries" most nearly means
47-64) is that A) information
A) the advent of search engines has transformed B) scheme
people's daily cycles. C) tickets
B) search entries can yield a fairly accurate D) routes
prediction of people's behaviors.
C) people generally use search engines to improve
their quality of life.
D) Germany was the first country to revolutionize
navigation technologies.

15
Which choice provides the best evidence for the
answer to the previous question?
A) Lines 48-51 ("If they...the bus")
B) Lines 54-57 ("The core...traffic")
C) Lines 58-59 ("Two...reports") 17
D) Lines 59-62 ("Similarly...reports")
As used in line 42, "deploying" most nearly means
A) utilizing
B) sending
C) expanding
D) displaying

3 3

U n
a
u
t
h
o
r
i
z
e
d
c
o
p
y
i
n
g
o
r
r
e
u
s
e
o
f
a
n
y
p
a
r
t
o
f
t
h
i
s
p
a
g
e
i
s
i
l
l
e
g
a
l
. 8

10
31
18 20
Which choice best supports the conclusion that using Taken together, the two figures suggest that google
search entries for traffic jams might not be used in searches
the future? A) closely match the actual traffic intensity.
A) line 65-69 ("A future...method") B) accurately capture the actual traffic intensity in
B) line 69-73 ("But currently...drive") the morning.
C) line 73-76 ("To my...conditions") C) fail to predict the actual traffic intensity.
D) line 76-79 ("It does... congestion") D) show a different timeline for the actual traffic
intensity.

19
Which choice is supported by the data in the first
figure?
A) Traffic reaches its lowest level at around 2 PM
in Cleveland, Ohio.
B) Traffic reaches its highest level at around 5 PM
in Cleveland, Ohio.
C) Traffic reaches its highest level at around 7 AM
in Cleveland, Ohio.
D) Traffic levels are about the same at 7 AM and
5 PM in Cleveland Ohio.

3 3

U n
a
u
t
h
o
r
i
z
e
d
c
o
p
y
i
n
g
o
r
r
e
u
s
e
o
f
a
n
y
p
a
r
t
o
f
t
h
i
s
p
a
g
e
i
s
i
l
l
e
g
a
l
. 9

11
32
Q u estion s 21-3 0 are b ased on th e f
o l
low ing evidence, several trees that had recently been stripped of
p a ss age. bark were discovered in the area. When the leaves are
45down in winter, a drone could be used to rapidly survey a
T h i
s pas
sag e i
s a d a
p t
ed from " U s
ing a Do n
re to Search for
vast tract of hardwoods for such foraging sites. This
th e Iv
ory-Bi
lled Wo o
dpecker",by M i
chae lDC .
o l
li
ns, 2018.
approach could be used to assess an area for the presence
The ivory-billed woodpecker is an exceptionally elusive of the ivory-billed woodpecker prior to beginning a
species that has been thought to be extinct only to be dedicated search effort. A drone is capable of providing
rediscovered several times during the past hundred years. 50detailed images of individual trees, which could reveal
The most recent rediscovery in Arkansas motivated searches
L
i
n
e foraging patterns that seem to be inconsistent with other
in Florida and Louisiana, where additional sightings were
5 woodpeckers and/or are suggestive of the ivory-billed
reported. But the issue became uncertain when nobody woodpecker.
managed to obtain a clear photo, which is regarded as the The 94 degree lens of the video camera covers a swath
standard form of evidence for documenting birds. An 55that is about 80 m wide from an altitude of 40 m. Four
analysis based on factors related to habitat and behavior square kilometers of habitat could therefore be covered with
suggests that the expected waiting time for obtaining such
1
0 ten flights of 5 km, which could be done in about two
evidence for the persistence of the ivory-billed woodpecker hours at 8 m/s. On the ground, it can take a few hours to
is several orders of magnitude greater than it would be for follow a search path for just a few kilometers through the
a more typical baseline species of comparable rarity. These 60dense vegetation and flooded areas in such habitats. It
factors include the wariness of the ivory-billed woodpecker would take only a few days for a small team of drone
(which is documented in historical accounts), the vastness of
1
5 operators to search tens of square kilometers for signs of
its habitats (typically covering several tens of square foraging and assess the area for the presence of ivory-billed
kilometers), the difficulty of moving along a search path in woodpeckers. When there are indications that ivory-billed
its habitats (which contain thick vegetation, waterways and 65woodpeckers are present, dedicating resources to a large
flooded areas) and the lack of visibility in a forest (typically number of search missions might eventually pay off with a
limited to a few tens of meters).
2
0 favorable encounter with ivory-billed woodpeckers. In order
During searches that were carried out in the previous to fully exploit the capability of a drone for this
decade, attempts to document this species involved strategies application, one would need to acquire a permit to
such as drifting through appropriate habitat in a kayak, 70fly missions beyond the line of sight over the remote
observing from a vantage point that provides a view out and uninhabited habitats that are appropriate for
over the treetops and deploying autonomous cameras and
2
5 ivory-billed woodpeckers. Another possible approach
audio recorders. The feasibility of searching for the would be to launch missions from vantage points that
ivory-billed woodpecker and surveying its habitat with a provide views out to long ranges. There are many
drone is explored here. With this approach, it is possible to 75trees within the main study area that could serve this
cover a much larger area per unit time than can be covered purpose, including some that have been used as
by a single person on the ground. A drone provides an
3
0 observation platforms.
unobstructed view when the camera is aimed over the
treetops, and the view of the dorsal field marks of a bird in
flight is favorable when the camera is aimed downward. By
tilting the camera slightly forward from vertical, it may be
possible to obtain video footage of a wary bird that flushes
3
5
as the drone approaches.
One of the potential uses of a drone would be to search
for signs of foraging that are consistent with the ivory-billed
woodpecker, which has a massive bill that gives it an
advantage over other woodpeckers within its range in
4
0
accessing rich food sources beneath tightly adhered bark.
Shortly before a sighting in 2008 that is supported by video

U n
a
u
t
h
o
r
i
z
e
d
c
o
p
y
i
n
g
o
r
r
e
u
s
e
o
f
a
n
y
p
a
r
t
o
f
t
h
i
s
p
a
g
e
i
s
i
l
l
e
g
a
l
. 1
0

12
33
21 24
Which choice best reflects the overall sequence of Which choice provides the best evidence for the
events in the passage? answer to the previous question?
A) A study is presented and explained, and it is A) Lines 1-3(“The ivory-billed...years”)
modified to test a new hypothesis. B) Lines 4-6(“The most...reported”)
B) An experiment is conducted, and its results and C) Lines 6-8(“But...birds”)
implications are analyzed. D) Lines 8-13(“An analysis...rarity”)
C) Previous researches are listed, and their
procedures and results are carefully evaluated.
D) Difficulties in resolving a problem are listed, and
a particular solution is advocated.

25
22
In the second paragraph (lines 21-36), the
As used in line 4, "motivated" most nearly means explanation about a drone mainly serves to
A) swayed A) present the advantages of using a drone for
B) triggered documentation.
C) excited B) compare previous filming strategies to current
D) persuaded ones.
C) emphasize the vast amount of land that a drone
can fly over.
D) list various ways in which a drone tilts a
camera to film a woodpecker.

23
Which statement best summarizes the reason for the
uncertainty surrounding the continued existence of
ivory-billed woodpeckers?
A) Current technologies in photography are not
developed enough for bird documentation.
B) Ivory-billed woodpeckers move at a speed that
does not allow researchers to capture them.
C) Documenting ivory-billed woodpeckers usually
requires exceptional effort.
D) Researches on ivory-billed woodpeckers have
been suspended for an excessive amount of time.
3

U n
a
u
t
h
o
r
i
z
e
d
c
o
p
y
i
n
g
o
r
r
e
u
s
e
o
f
a
n
y
p
a
r
t
o
f
t
h
i
s
p
a
g
e
i
s
i
l
l
e
g
a
l
. 1
1

13
34
26 29
The sighting of "several trees that had recently been What can be reasonably inferred about a drone from
stripped of bark" might indicate the presence of the passage?
ivory-billed woodpeckers because A) A drone can survey a vast area of land without
A) they are comparably more effective at foraging damaging the environment.
than other birds are. B) A drone can cover a large amount of land
B) they habitually tear away tree barks to mark quickly.
food sources. C) Using a drone can eliminate the need for a
C) they can use its large bills for foraging video camera.
underneath tree barks D) A drone can automatically identify ivory-billed
D) only they can perform the action of foraging on woodpeckers.
trees.

27 30
The passage implies which of the following as a Which choice provides the best evidence for the
reason that a drone is used to survey tree barks? answer to the previous question?
A) Ivory-billed woodpeckers' foraging patterns are A) Lines 49-53 ("A...woodpecker")
identifiable through images provided by a drone. B) Lines 60-64 ("It...woodpeckers")
B) Ivory-billed woodpeckers presence can only be C) Lines 64-67 ("When...woodpeckers")
found through investigation using a drone. D) Lines 72-74 ("Another...ranges")
C) Ivory-billed woodpeckers do not have the
capacity to detect the presence of a drone.
D) Ivory-billed woodpeckers are not wary enough to
avoid a surveying drone.

28
As used in line 63, "assess" most nearly means
A) calculate
B) estimate
C) check
D) compute

U n
a
u
t
h
o
r
i
z
e
d
c
o
p
y
i
n
g
o
r
r
e
u
s
e
o
f
a
n
y
p
a
r
t
o
f
t
h
i
s
p
a
g
e
i
s
i
l
l
e
g
a
l
. 1
2

14
35
Q u estio ns 3 1-41 are b ase d o n th e fo l
lo w ing Pa ssage 2
p a ssa g es. What good does it do as a measure of reform if
P as sa
g e 1 is ad apted from a speech give n by Senato r Hwa e
yl
45the power of removal is unlimited and without cause
o n Dc em b er 13 , 1
882 . Passag e2 is ad apte d f r
om a speec h at the mere will or whim of the appointing power?
g ive n by Sen ator Brow n on Dc em b er 1 4, 188 2. The civil service reformers who are most clamorous
for action, and who are in earnest about the matter,
P a ssag e 1 would consider such a measure, if that is all it
There has been a constant and a steady growth of the idea 50means, as a trick, a sham, a delusion.
that offices might be used to strengthen candidates and to reward But it requires a competitive examination, say the
active workers. The doctrine that "to the victors belong the Senators on the other side, before you put a man
spoils" became (though understandable in our two-party system)
L
i
n
e into office. Then again the bill is a cheat and a
5
the practical rule of the country. The evils of the existing system mockery. It does no such thing in spirit and
can not be denied by any man, whatever his party or his position 55substance. For fear there might come a day when a
with regard to any of the pending measures for civil-service Democratic Executive would administer the affairs of
reform. They are most clearly obvious to members of Congress this Government, and that day might not be very
than to anybody else. They are obvious in the suffering and distant, there is a careful provision in this bill that it
1
0
humiliation of the employees. The condition of the majority of shall apply only to the lowest class who are to hold
them is pitiable. They are under a sort of degradation that we 60office. There shall be a competitive examination for
have no right to impose upon our friends and neighbors and the lowest grade only; that is free to all: and the
fellow-citizens. They are only partially set in their positions by Senator from Massachusetts who took his seat a few
their character and by the good work they may do. How well we minutes ago very earnestly stated that that was one
know that they do not depend upon those things to maintain
1
5 of the strong features of the bill.
them in place; that they are constantly coming to members of 65 Now, I believe that there is a very large number
Congress and applying to influential friends everywhere to of employees in the Departments at present,
strengthen what they call their "influence" till the word occupying different positions in them, some of them
"influence" has become a cant term, a slang term among them. high positions, who are not fit for those places,
2
0
"Who is your influence?" is the phrase. "I have none. My morally, intellectually, or in any other fashion; but
influence is dead." Or, "My influence was in Congress ten or 70the charmed circle is not to be disturbed. If there
fifteen years ago, and he is not in political life now, or he has chances to be one of the lower clerkships vacant,
no influence himself;" "I must get some influence," et cetera. then the doors are thrown wide open by this bill and
These are the every-day phrases among the employees; and every American citizen may come up and compete
whenever a new chief of a bureau comes in, not to say a new
2
5 for it. It will not do to go higher than that, for too
Cabinet officer or a new President, there is a hurrying and a 75many Democrats might get in. You Democrats can
scurrying among all the terrified flock to strengthen themselves in come up and compete for the lowest clerkships that
position; not by the good record they may have or the good are to be filled; but if a vacancy occurs above that,
character they may have maintained, but by the recommendations then the Republican employees and officers already in
of political friends. By this system the inefficient are kept in
3
0 office, and they alone, can apply for the advancement
longer than they would be otherwise. These are facts so well 80or promotion. That is the civil-service reform that
known that I ought to ask pardon for repeating them. this bill gives to the country; that is the share that
The man; who is less efficient than his fellows, conscious that the Democratic party gets in it. I repeat it, under the
he has less of character or of ability, or of both, than they, is provisions of this bill the competition is only general
3
5
the man, who is almost certain to have the largest pile of papers for the lowest office that can become vacant. There a
in support of his position. And thereby it becomes exceedingly 85Democrat stands a chance to get in this lowest
difficult to remove him. More persons are needed for the same position, but if fifty vacancies occur above it only
labor than there would be under some ideal system, I do not say the present incumbents, the Republican office-holders,
what. We can imagine that if they were appointed purely by can compete for the promotion. To the victors belong
ability and maintained for that, fewer persons — I do not pretend
4
0 the spoils. Thus our paltry share in its benefits.
to say how many, because no man knows; the estimate are quite
at random; some say a quarter less, some say a half — would do
the work equally well.

U n
a
u
t
h
o
r
i
z
e
d
c
o
p
y
i
n
g
o
r
r
e
u
s
e
o
f
a
n
y
p
a
r
t
o
f
t
h
i
s
p
a
g
e
i
s
i
l
l
e
g
a
l
. 1
3

15
36
31 34
As used in line 13, "set in" most nearly means According to the author of Passage 1, in order for
A) appointed to efficiency in the bureaucracy to increase, the country
B) safe upon must
C) assured at A) increase the number of offices and
D) tightened around corresponding positions in the bureaucracy.
B) downsize the bureaucracy to only include
those who are talented.
C) open up the upper positions in the
bureaucracy even to those not from the
ruling party.
D) revolutionize the way bureaucracy functions
in relation to the Congress.

32
It can be inferred that the author of Passage 1 35
believes that those without good character and ability
A) are doomed to fail and become dishonorably As used in line 69, "fashion" most nearly means
discharged from office. A) matter
B) are mostly non-influential or retired members B) trend
of the Congress. C) manner
C) cannot be removed from their office without D) pattern
first removing Congressional support.
D) can still cling to their position with ample
amount of influence.

33
Which choice provides the best evidence for the
answer to the previous question?
A) Lines 14-19 ("How...them.")
B) Lines 24-28 ("These...position.")
C) Lines 30-32 ("By this...them.")
D) Lines 33-37 ("The man...him.")

U n
a
u
t
h
o
r
i
z
e
d
c
o
p
y
i
n
g
o
r
r
e
u
s
e
o
f
a
n
y
p
a
r
t
o
f
t
h
i
s
p
a
g
e
i
s
i
l
l
e
g
a
l
. 1
4

16
37
36 39
In passage 2, the author claims that the reform would Which best describes the overall relationship
be truly effective only if between Passage 1 and Passage 2?
A) the Republicans open up more Senate A) Passage 2 directly objects to implementing
positions to the Democrats. the bill proposed by Passage 1.
B) it allows for even incompetent high officers to B) Passage 2 elaborates on the conditions
be reshuffled. necessary to realize the conclusion of Passage 1.
C) the current bureaucracy is completely replaced C) Passage 2 agrees in principle with the
by a new one. argument set forth by Passage 1.
D) people who wield influence do not establish D) Passage 2 doubts the feasibility of the
any relations with those below. outcome anticipated by passage 1.

37 40
Which choice provides the best evidence for the The authors of both passages would most likely
answer to the previous question? agree with which of the following statements about
A) Lines 47-50 ("The civil...delusion.") the bureaucracy?
B) Lines 53-60 ("Then...office.") A) the Republican party would retain high
C) Lines 61-64 ("and the...bill.") positions in the bureaucracy even after the
D) Lines 75-80 ("You...promotion.") reform.
B) the influence exerted on it by the Congress
will be largely curtailed in the future.
C) the corruption within the lower levels at the
bureaucracy would be cleansed of corruption
when the bill is passed.
D) the reform cannot be implemented without
the support from bureaucrats in high positions.
38 41
In lines 88-89 (“To the… spoils”), the author of Which of the following best represents how the
Passage 2 refers to a statement made in Passage 1 in author of Passage 1 would most likely respond to
order to the points made in the final paragraph (lines 65-89)
A) validate his own reasoning about the need to of Passage 2?
remove the Republicans from power. A) Lines 3-5 (“The doctrine…country”)
B) express desire to establish rapport on the
B) Lines 8-9 (“They…else”)
reform bill with Hawley. C) Lines 20-23 (“Who…et cetera”)
C) question Hawley's intention regarding the D) Lines 37-39 (“More…what”)
present debacle.
D) establish common ground between his argument
and that of Hawley

U n
a
u
t
h
o
r
i
z
e
d
c
o
p
y
i
n
g
o
r
r
e
u
s
e
o
f
a
n
y
p
a
r
t
o
f
t
h
i
s
p
a
g
e
i
s
i
l
l
e
g
a
l
. 1
5

17
38
Q u estions 42 -
52 a re b ased on the f
o ll
ow ing ingredient resists gastric acidity and absorption in the
passage and supplementary materials upper gastrointestinal tract, is fermented by the
45intestinal microflora, and selectively stimulates the
T his pass
ag e isan excerpt f
rom " Fi
be rand Prebioti
cs:
growth and/or activity of intestinal bacteria associated
M e chanis
m s and Ha l
e
th Bene f
its
",b y J
oanne Slavi
n ,
with health and well-being. Prebiotics occur naturally
2 013.
in foods such as leeks, asparagus, chicory, Jerusalem
The term “dietary fiber” was coined in 1953, but artichokes, garlic, onions, wheat, oats, and soybeans.
the health benefits of high fiber foods have been long 50 The concept of prebiotics is relatively new;
appreciated. In 430 BC, Hippocrates described the nutritionist Marcel Roberfroid introduced the concept
laxative effects of coarse wheat in comparison with
L
i
n
e in 1997. However, foods high in prebiotics have been
refined wheat. In the 1920s, J.H. Kellogg published
5 consumed since prehistoric times. Archaeological
extensively on the attributes of bran, claiming it evidence from dry cave deposits in the northern
increased stool weight, promoted laxation, and 55Chihuahuan Desert show intensive utilization of desert
prevented disease. Dietary fiber was researched plants that were high in inulin. Analysis of
throughout the 1930s, and then forgotten until the well-preserved coprolites suggest that dietary intake of
1970s.
1
0 inulin was about 135 g/day for the typical adult male
Denis Burkitt is usually credited with hunter-forager. By comparison, consumption in typical
re-popularizing the idea that dietary fiber protects 60US and European diets today has been estimated to
against development of Western diseases, including be 4-5 grams per day.
diabetes, cardiovascular disease, colon cancer, and An important mechanism of action for dietary
obesity.
1
5 Since that time, research continued on fiber and prebiotics is fermentation in the large
defining fiber, measuring fiber, and determining the intestine and changes in gut microflora. The human
health benefits of fiber consumption. Prospective, 65large intestine is one of the most diversely colonized
cohort studies show clearly that high fiber intakes are and metabolically active organs in the human body.
linked to less cardiovascular disease. Data on obesity, Up to 1,000 different species of bacteria reside in the
diabetes, and cancer are more mixed. Dietary fiber is
2
0 large intestine with microbial populations comprising
listed on the Nutrition Facts panel on food products approximately 1011–1012 cfu/g of contents. The
and nutrient content claims are allowed for good (2.5 70colonic environment is favorable for bacterial growth
g fiber) and excellent (5.0 g fiber) sources of fiber. due to its slow transit time, readily available
Prebiotics were first defined as “nondigestible food nutrients, and favorable pH. Generally, bacteria having
ingredients that benefit the host by selectively
2
5 an almost exclusive saccharolytic metabolism (i.e., no
stimulating the growth and/or activity of one or a proteolytic activity) can be considered potentially
limited number of bacteria in the colon, thus 75beneficial.
improving host health”. This definition was later Dietary fibers exhibit a diverse range of
refined to include other areas that may benefit from physiochemical properties and corresponding
selective targeting of particular microorganisms: “a
3
0 physiological effects. The role of fiber in health has
selectively fermented ingredient that allows specific extended far beyond improved laxation, and includes
changes, both in the composition and/or activity in the 80benefits on risk factors for cardiovascular disease,
gastrointestinal microflora, that confer benefits.” weight management, immune function, and colonic
Lactobacilli and bifidobacteria are the usual target health. However, it is clear that not all fibers are
genera for prebiotics; changes in bifidobacteria are
3
5 equal in terms of the types and extent of health
more likely to be seen compared to lactobacilli. This benefits they provide. Characteristics such as
may be due to the fact that more bifidobacteria 85solubility, fermentability, and viscosity are important
usually reside in the human colon than lactobacilli, determinants of the effect the fiber will have in the
and they exhibit a preference for oligosaccharides. body. Fibers with prebiotic properties can also be
4
0 Although all prebiotics are fiber, not all fiber is recommended as part of fiber intake, although studies
prebiotic. Classification of a food ingredient as a are lacking on the benefits of prebiotic intake to
prebiotic requires scientific demonstration that the

U n
a
u
t
h
o
r
i
z
e
d
c
o
p
y
i
n
g
o
r
r
e
u
s
e
o
f
a
n
y
p
a
r
t
o
f
t
h
i
s
p
a
g
e
i
s
i
l
l
e
g
a
l
. 1
6

18
39
healthy individuals. Due to the variability of fiber’s
90
42
effects in the body, it is important to consume fiber
from a variety of sources. Since fiber intakes around The author mentions "Hippocrates" and "Kellogg" in
the world are less than half of recommended levels, the first paragraph lines (1-10) in order to
increasing fiber consumption for health promotion and A) lament the fact that people had forgotten
disease prevention is a critical public health goal.
95 high fiber foods in 1970s.
B) indicate that people have long been aware of
Types of Dietary Fiber the benefits of high fiber foods.
C) praise the figures who succeeded in
popularizing high fiber foods.
D) name scholars who specialized in researching
high fiber foods.

Ch
a
r
ta
d
a
pm F
t
e
d
f
r
o i
be
randP
reb
i
o
t
i
c
s
:M e
c
h
a
n
i
s
m s
an
d
Ha
l
e
t
h
Be
n
e
f
i
t
s
b
yJo
an
ne
S
lav
in,
2
013 43
In line x, the author states that Denis Burkitt was
responsible for making fiber popular again. Based
on the passage, which choice best describes Burkitt's
contribution to the topic of dietary fiber?
A) He designed the food pyramid that
contributed to the improvement of citizens'
health.
B) He specified the amount of dietary fiber
intake necessary for human body.
C) He triggered research that assessed the health
benefits and characteristics of fiber.
D) He helped develop dietary fiber that was more
suitable for consumption

44
Which choice provides the best evidence for the
answer to the previous question?
A) Lines 11-15 (“Denis Burkitt...obesity”)
B) Lines 15-17 (“Since...consumption”)
C) Lines 17-19 (“Prospective...disease”)
D) Lines 20-23 (“Dietary fiber...fiber”)

U n
a
u
t
h
o
r
i
z
e
d
c
o
p
y
i
n
g
o
r
r
e
u
s
e
o
f
a
n
y
p
a
r
t
o
f
t
h
i
s
p
a
g
e
i
s
i
l
l
e
g
a
l
. 1
7

19
40
45 47
As used in line 39, “exhibit” most nearly means to Which choice provides the best evidence for the
A) draw answer to the previous question?
B) demonstrate A) Lines 47-49 (“Prebiotics...soybeans”)
C) perform B) Lines 50-52 (“The concept...1997”)
D) advertise C) Lines 52-53 (“However...times”)
D) Lines 56-61 (“Analysis...per day”)

46 48
The passage most strongly suggests that prehistoric The main purpose of the fourth paragraph (lines
people 40-49) is to
A) lived in a cave in order to cultivate and A) list the categories that qualify fiber as
consume prebiotics prebiotics
B) consumed more prebiotics than contemporary B) provide details about fiber's activities in
people human body
C) were the first ones to write about prebiotics C) introduce the benefits of consuming prebiotics
consumption D) present information on nutrients that compose
D) lacked food resources other than fibers that prebiotics
contain prebiotics

3 3

U n
a
u
t
h
o
r
i
z
e
d
c
o
p
y
i
n
g
o
r
r
e
u
s
e
o
f
a
n
y
p
a
r
t
o
f
t
h
i
s
p
a
g
e
i
s
i
l
l
e
g
a
l
. 1
8

20
41
49 51
An assumption made by the author in the last Which statement about dietary fiber is best
paragraph is that fibers supported by the table and the passage?
A) are all highly soluble and fermentable and A) Most Americans do not consume enough
facilitate digestion. dietary fiber, meeting only half of their daily
B) are not necessary because of today's food recommended levels.
processing technologies. B) The types of dietary fiber vary depending on
C) have different qualities that determine health solubility, fermentability, and viscosity.
benefits they provide. C) Viscous dietary fibers are the healthiest form
D) are promoted by government policies that of fiber, closely followed by fermentable,
encourage fiber consumption. non-viscous fibers.
D) Resistant starches can be both fermentable and
non-fermentable, but neither type is beneficial
for the body.

50
Based on data in the table, which choice is a
soluble, fermentable, and non-viscous dietary fiber?
A) Inulin
B) Cellulose
C) Pectin
D) Lignin
52
What information, if added to the table, would be
helpful in determining which types of fiber to
consume?
A) The different health benefits provided by
each type of dietary fiber.
B) Whether or not each type of dietary fiber is
a prebiotic.
C) The recommended daily dosage of dietary
fiber and a list of foods rich in fiber.
D) A detailed categorization of bacteria living in
the colon.

3 3

U n
a
u
t
h
o
r
i
z
e
d
c
o
p
y
i
n
g
o
r
r
e
u
s
e
o
f
a
n
y
p
a
r
t
o
f
t
h
i
s
p
a
g
e
i
s
i
l
l
e
g
a
l
. 1
9

21
42
SAT
Reading Test 2

22
22
23
1
Reading Test
65 MINUTES, 52 QUESTIONS

Turn to Section 1 of your answer sheet to answer the questions in this section.

DIRECTIONS

Each passage or pair of passages below is followed by a number of questions. After reading each
passage or pair, choose the best answer to each question based on what is stated or implied in the
passage or passages and in any accompanying graphics (such as a table or graph).

Questions 1-11 are based on the following passage. a lowly clerk, but they needed him all the same.
Marc much preferred his office, despite the
The following passage is adapted from Patrick 25 profound lack of excitement of his work, to his
Waddington, The Street That Got Mislaid, © Patrick room on Oven Street (running north and south
Waddington, 1954. from Sherbrooke East to St. Catherine), where his
neighbors were noisy and sometimes violent, and
Marc Girondin had worked in the filing his landlady consistently so. He tried to explain
section of the city hall’s engineering department 30 the meaning of his existence to a fellow tenant,
for so long that the city was laid out in his mind Louis, but without much success. Louis, when he
Line like a map, full of names and places, intersecting got the drift, was apt to sneer.
5 streets and streets that led nowhere, blind alleys “So Craig latches on to Bleury and Bleury gets
and winding lanes. to be Park, so who cares? Why the excitement?”
In all Montreal no one possessed such 35 “I will show you,” said Marc. “Tell me, first,
knowledge; a dozen policemen and taxi drivers where you live.”
together could not rival him. That is not to say “Are you crazy? Here on Oven Street. Where
10 that he actually knew the streets whose names he else?”
could recite like a series of incantations, for he “How do you know?”
did little walking. He knew simply of their 40 “How do I know? I’m here, ain’t I? I pay my
existence, where they were, and in what relation rent, don’t I? I get my mail here, don’t I?”
they stood to others. Marc shook his head patiently.
15 But it was enough to make him a specialist. He “None of that is evidence,” he said. “You live
was undisputed expert of the filing cabinets here on Oven Street because it says so in my
where all the particulars of all the streets from 45 filing cabinet at city hall. The post office sends
Abbott to Zotique were indexed, back, forward you mail because my card index tells it to. If my
and across. Those aristocrats, the engineers, the cards didn’t say so, you wouldn’t exist and Oven
20 inspectors of water mains and the like, all came to Street wouldn’t either. That, my friend, is the
him when they wanted some little particular, triumph of bureaucracy.”
some detail, in a hurry. They might despise him as 50 Louis walked away in disgust. “Try telling that

Unauthorized copying or reuse of any part of this page is illegal.


24
2 CONTINUE
1
to the landlady,” he muttered. 1
So Marc continued on his undistinguished Over the course of the passage, the main focus
career, his fortieth birthday came and went shifts from
without remark, day after day passed A) an explanation of a city’s layout to a
55 uneventfully. A street was renamed, another description of one particular street.
constructed, a third widened; it all went carefully B) the description of a character’s personality
into the files, back, forward and across. to the introduction of a life-changing
And then something happened that filled him event.
with amazement, shocked him beyond measure, C) an interaction between two characters to
60 and made the world of the filing cabinets tremble the effects of that interaction on one of
to their steel bases. them.
One August afternoon, opening a drawer to its D) a character’s thoughts to the depiction of
fullest extent, he felt something catch. Exploring him acting upon those thoughts.
farther, he discovered a card stuck at the back
65 between the top and bottom. He drew it out and 2
found it to be an old index card, dirty and torn, The imagery in lines 3-6 (“city ... lanes”)
but still perfectly decipherable. It was labeled primarily serves to
RUE DE LA BOUTEILLE VERTE, or GREEN A) characterize the mysterious nature of
BOTTLE STREET. Montreal.
70 Marc stared at it in wonder. He had never B) demonstrate the intricacy of Marc’s
heard of the place or of anything resembling so memory.
odd a name. Undoubtedly it had been retitled in C) show that nobody else could understand
some other fashion befitting the modern the map.
tendency. He checked the listed details and D) emphasize Marc’s exceptional sense of
75 ruffled confidently through the master file of direction.
street names. It was not there. He made another
search, careful and protracted, through the 3
cabinets. There was nothing. Absolutely nothing. The situation described in paragraph two
Once more he examined the card. There was no (lines 7-14) is most like that of
80 mistake. The date of the last regular street A) a cook who is an expert on a specific dish
inspection was exactly fifteen years, five months but who has never tasted it.
and fourteen days ago. B) a travel agent who makes a living
As the awful truth burst upon him, Marc describing destinations to clients.
dropped the card in horror, then pounced on it C) a librarian who catalogues and organizes
85 again fearfully, glancing over his shoulder as he books for the general public.
did so. D) a student who is interested in a particular
It was a lost, a forgotten street. For fifteen artist but never seen any of her work.
years and more it had existed in the heart of
Montreal, not half a mile from city hall, and no
90 one had known. It had simply dropped out of
sight, a stone in water.

Unauthorized copying or reuse of any part of this page is illegal.


25
3 CONTINUE
1
4 9
Which statement best characterizes Marc’s At the end of the passage, the narrator implies
relationship with his job? that Marc is
A) He enjoys his job but dislikes certain tasks A) confused as to why he forgot to file a
he must perform. particular street.
B) He is good at his job but does not enjoy it. B) curious about what the street contains.
C) He is consumed by his job and believes it to C) anxious about the quality of the street
be all-important. inspections.
D) He finds his job fun but does not realize D) terrified by the discovery he has made.
how significant his work is.
10
5 Which choice provides the best evidence for
Based on the passage, the other workers in the the answer to the previous question?
engineering department view Marc as A) lines 76-78 (“It ... nothing”)
A) valuable but looked-down-upon. B) lines 80-82 (“The date ... ago”)
B) intelligent but underused. C) lines 83-86 (“As ... so”)
C) superior but shy. D) line 87 (“It ... street”)
D) inferior but quick.
11
6 Lines 87-91 (“It was ... water”) mainly serve to
It can be reasonably inferred that Marc and A) highlight the importance of a character’s
Louis differ primarily because discovery.
A) Marc is poetic and Louis is unimaginative. B) illustrate the main character’s unusual
B) Marc is delusional and Louis is realistic. point of view.
C) Marc is productive and Louis is lazy. C) describe a journey taken by the main
D) Marc is kind and Louis is brusque. character.
D) depict a new setting that a character will
7 explore.
Which choice provides the best evidence for
the answer to the previous question?
A) lines 40-43 (“How ... said”)
B) lines 43-45 (“You ... hall”)
C) lines 45-46 (“The post ... to”)
D) lines 46-51 (“If ... muttered”)

8
As used in line 63, “exploring” most nearly
means
A) traveling.
B) inquiring.
C) examining.
D) evaluating.

Unauthorized copying or reuse of any part of this page is illegal.


26
4 CONTINUE
1
Questions 12-21 are based on the following Virgo. Together we will travel through the
passage. expanding universe, and we had better learn to
like the company. [A]ny galaxies that have not yet
Passage 1 is excerpted from Tamara Davis, “If Galaxies are 35 won the gravity war have missed their chance.
All Moving Apart, How Can They Collide?” ©2009 by The universe is now split into pockets of
Scientific American. Passage 2 is excerpted from David interaction that will drift alone through the
Biello, “More Often Than Not, Massive Galaxies Form by expanding cosmos.
Mergers” ©2005 by Scientific American. Like revelers on a ship, the galaxies in our
40 group will continue to collide and interact in
Passage 1 myriad interesting ways, but we will be forever
Dark energy, believed to be causing the separated from the revelers on other ships sailing
acceleration of the expansion of the universe, away from us in the vast universe.
provides a constant outward force that does not
Line dilute as the universe expands. Pitted against this Passage 2
5 relentless push is the gravitational pull from the New data seem to show that galaxies collide
rest of the matter and energy in the universe. 45 all the time. In fact, the oldest and largest
Early on, the universe was much denser than it is galaxies in the universe most likely formed from
today, and the attractive force of gravity was such intergalactic combinations.
winning the battle, on scales both large and small. Astronomer Pieter van Dokkum of Yale
10 Clouds of gas condensed to form stars and University used some of the longest and deepest
galaxies, and galaxies drew together to form 50 sky surveys ever conducted to try to determine
clusters. If there had been more matter around, whether the oldest, largest galaxies— called
the universe might have started to re-collapse ellipticals because they lack the swirling arms of
before it ever had the chance to accelerate. But the spiral type, like our own Milky Way--formed
15 matter and energy do dilute as the volume of the from the collapse of ancient clouds of gas or the
universe increases, so dark energy slowly came to 55 accretion of smaller galaxies bumping into each
dominate. Since about six billion years ago (about other. Of the 126 galaxies of all varieties van
a billion years before Earth formed), the Dokkum looked at, 67 showed telltale signs of
expansion has, on average, been accelerating. impact, such as trailing tails of stars, or a collision
20 Nevertheless, the cosmic dance continues. in progress.
Galaxies that had been pulled together before the 60 “Our study found these common massive
universe began accelerating still have the chance galaxies do form by mergers,” Van Dokkum
to collide. Collectively, they form over-dense explains. “It is just that the mergers happen
patches of the universe in which gravity still quickly and the features that reveal the mergers
25 reigns. In our neighborhood the Andromeda are very faint and therefore difficult to detect.”
galaxy, our largest companion, is actually falling 65 “Quickly” on a galactic scale means just a few
toward us, and we will have our first close hundred million years--a small fraction of the
encounter with it in just a few billion years' time. 13.7 billion years the universe has been in
Our local group comprises Andromeda, the existence--and, because such collisions rarely
30 Magellanic Clouds and about 35 other galaxies, involve head-to-head star crashes, they leave few
all of which lie in an even larger cluster called 70 traces behind except in the shape of the resulting

Unauthorized copying or reuse of any part of this page is illegal.


27
5 CONTINUE
1
galaxy and a general slowing in its formation of 12
new stars. Based on Passage 1, which of the following
None of the six spiral galaxies in the survey choices suggests that the expansion of the
showed any after-crash damage, but that doesn't universe is accelerating?
75 mean that our own galaxy is free and clear. “The A) New galaxies are continuously forming.
Milky Way will indeed undergo a collision in the B) Galaxy clusters are moving further apart.
near future as we are heading toward M31, the C) Ancient galaxies are gaining in mass.
Andromeda Nebula,” van Dokkum adds. “‘Near D) Galaxy collisions are occurring more
future’ in this case is about four billion years from frequently.
80 now though.”
13
Which choice provides the best evidence for
the answer to the previous question?
A) lines 14-17 (“But ... dominate”)
B) lines 21-23 (“Galaxies ... collide”)
C) lines 29-32 (“Our ... Virgo”)
D) lines 36-38 (“The universe ... cosmos”)

14
As used in line 8, “attractive” most nearly
means
A) advantageous.
B) compelling.
C) absorbing.
D) pulling.

15
The author of Passage 1 claims that the Milky
Way and the Andromeda Galaxy will collide
mainly because
A) they are bound together by gravity.
B) they show signs of impact from billions of
years ago.
C) one is much denser than the other.
D) one is a spiral galaxy and the other an
elliptical galaxy.

Unauthorized copying or reuse of any part of this page is illegal.


28
6 CONTINUE
1
16 20
In Passage 2, the situation described in The authors of both passages would agree that
paragraph 3 is most similar to intergalactic collisions
A) an animal behavior that is difficult to A) can be predicted with relative accuracy.
analyze because it varies within the B) are most common in the outer reaches of
species. the cosmos.
B) a natural phenomenon that is difficult to C) are inevitable within confined groups.
document because it is rarely seen. D) can be explained by the dominance of dark
C) a physical law that is logical in theory but matter.
still largely unfounded.
D) an artistic style that is gaining in 21
popularity but still widely unknown. Which type of evidence is used extensively in
passage 2 but not in passage 1?
17 A) Summaries of scholarly opinion
It can reasonably be inferred from Passage 2 B) References to historical sources
that collisions throughout the universe C) Judgments of renowned scientists
A) are largely undocumented. D) Results from a study
B) can cause clouds of collapsing gas.
C) occur at a relatively consistent rate.
D) rarely involve spiral galaxies.

18
Which choice provides the best evidence for
the answer to the previous question?
A) lines 45-47 (“The oldest ... combinations”)
B) lines 56-59 (“Of ... progress”)
C) lines 68-72 (“because ... new stars”)
D) lines 73-74 (“None ... damage”)

19
Compared with the tone of passage 2, the tone
of passage 1 is more
A) lyrical.
B) somber.
C) speculative.
D) technical.

Unauthorized copying or reuse of any part of this page is illegal.


29
7 CONTINUE
1
Questions 22-32 are based on the following playtime to overall activity requirements, with
passage and supplementary information. contributions being as low as 4.5%.
35 A number of studies have successfully
This passage is excerpted from Carly Wood, Valier Gladwell increased playtime PA through the introduction
and Jo Barton, “A Repeated Measures Experiment of School of interventions such as sports or games
Playing Environment to Increase Physical Activity and equipment, playground markings, fitness breaks
Enhance Self-Esteem in UK School Children.” © 2014 by and playground structures. However, these types
Wood et al. 40 of interventions tend to facilitate structured
rather than unstructured PA. Unstructured PA is
The health benefits of engaging in physical essential to childhood development and therefore
activity (PA) during childhood include enhanced needs to be encouraged during playtime.
fitness, cognitive function and bone health, Natural environments can encourage
Line reduced body fatness, motor skill development, 45 unstructured play and may therefore play a role
5 and favourable cardiovascular and metabolic in facilitating unstructured play during playtime.
disease risk profiles. Being active during Natural environments provide large open spaces
childhood can also improve self-esteem and which encourage individuals to be active, whilst
reduce symptoms of anxiety and depression. areas lacking nature may restrict PA due to
Participation in PA in youth is of great 50 limited space and parental fears over crime and
10 importance as PA may tract into adulthood where road traffic. Children report a preference for play
adequate levels of PA are protective against many in natural environments, with nature facilitating
chronic diseases. However, in the UK more imaginative and inventive play.
approximately 75% of boys and 80% of girls aged Furthermore, adolescents living in urban settings
5-10 years are not meeting the daily 55 with access to green spaces such as parks are
15 recommendation of 60 minutes of moderate to more likely to be physically active than their
vigorous physical activity... peers without park access, indicating that all
Unstructured play is also an essential part of forms of nature can be used as a tool for engaging
childhood which enables children to develop a youth in PA. Thus, if school playtime were
relationship with their surrounding environment 60 performed on the school field it is possible that
20 and enhances social skills, coordination and children’s PA levels would be increased. To date,
strength. Outdoor environments facilitate play there is a lack of data quantifying the impact of
and are associated with increased levels of PA. natural environments on levels of PA in children,
Thus, children should be provided with daily particularly within the school setting.
opportunities to play outdoors. The school 65 Performing PA in a natural environment
25 environment provides such an opportunity (“Green Exercise”) has also been demonstrated to
through the provision of playtime. Playtime provide improvements in self-esteem in adults,
normally takes place on the school playground whether participants are simply viewing scenes of
and lasts for at least one hour per day. However, nature or directly interacting with natural
universally playtime is reported to make 70 environments. Studies in adolescents and
30 relatively small contributions to children’s overall children suggest that Green Exercise has no such
daily activity requirements. In the UK, only one additive effect on self-esteem compared to
known study has reported the contribution of exercise in other environments. However, the only

Unauthorized copying or reuse of any part of this page is illegal.


30
8 CONTINUE
1
known study in children examined the impact of a 22
75 green playtime intervention consisting of The main purpose of the passage is to
orienteering.* The task-oriented, structured A) defend an unpopular belief.
nature of orienteering may not facilitate the B) confirm a previously untested hypothesis.
green exercise effect. Unstructured free play in a C) summarize various studies of a social
natural environment may allow greater condition.
80 interaction with the environment, thus benefiting D) debunk common misconceptions
self-esteem. surrounding an issue.

* A competitive sport in which runners have to find their way 23


across rough country with the aid of a map and a compass. As used in line 10, “tract” most nearly means
Increase in Self-Esteem Post-Play for Boys and A) carry.
Girls in Different Play Environments B) assign.
Boys Girls Total C) monitor.
0.9 D) linger.
as Measured by the Rosenbarg Self-Esteem Scale

0.8

0.7
24
The statistics about UK children (lines 12-16)
Increase in Self-Esteem

0.6
primarily serve to
0.5
A) encourage readers to become more active.
0.4
B) call attention to a particular situation.
0.3 C) transition to a discussion of unstructured
0.2 and structured play.
0.1 D) introduce an argument that the authors
0
will later contradict.
Playground Fleld Total
Play Type 25
The author implies that studies that have
The starting self-esteem score was 28.8, out of a possible 40. increased playtime physical activity are
A) accurate, because they depict the benefits
Adapted from from Carly Wood, Valier Gladwell and Jo Barton, of play across all social groups.
“A repeated Measures Experiment of School Playing B) correct, because they have allowed for the
Environment to Increase Physical Activity and Enhance purchase of playground structures and
Self-Esteem in UK School Children,” originally published in other equipment to help facilitate play.
2014. C) faulty, because they ignore the prevalence
of natural environments.
D) limited, because they use methods that
encourage a particular type of physical
activity.

Unauthorized copying or reuse of any part of this page is illegal.


31
9 CONTINUE
1
26 29
Which choice provides the best evidence for Which choice provides the best evidence for
the answer to the previous question? the answer to the previous question?
A) lines 35-39 (“A number ... structures”) A) lines 65-70 (“Performing ... environments”)
B) lines 39-41 (“However ... PA”) B) lines 70-73 (“Studies ... environments”)
C) lines 41-43 (“Unstructured ... playtime”) C) lines 76-78 (“The task-oriented ... effect”)
D) lines 44-46 (“Natural ... playtime”) D) lines 78-81 (“Unstructured ... self-esteem”)

27 30
What does the author claim about “Green Which of the author’s claims about self-
Exercise” (line 66)? esteem does the chart best support?
A) It benefits adults’ self-esteem but its A) Unstructured PA in a natural environment
effects on children have not been fully may benefit self-esteem in children.
determined. B) Boys’ self-esteem benefits less than girls’
B) Looking at images of natural environments from PA only in structured settings.
is the best way to improve self-esteem. C) Studies of orienteering do not provide
C) Adolescents may benefit from green enough information to study effects on
exercise in the same way that adults do. self-esteem.
D) Self-esteem can only rise through direct D) Structured PA on playgrounds provides
interaction with the environment. greater benefits to self-esteem than other
activities do.
28
What does the author imply about physical 31
activity in a natural environment? It can be reasonably inferred from the chart
A) Scientific studies have determined that that
physical activity can only benefit self- A) Girls enjoyed playing more than boys.
esteem in adults. B) Playgrounds and fields are the only two
B) The relationship between physical activity viable options for physical activity in
in nature and self-esteem requires further children.
study. C) Boys are equally happy playing anywhere.
C) Although it has been studied, the effects of D) PA on playgrounds increased average
physical activity in a natural environment self-esteem more than PA in fields.
are unclear in both children and adults.
D) Physical activity in a natural environment 32
benefits self-esteem in both children and The difference between the average increase
adults. in girls’ self-esteem and boys’ self-esteem
post-play is closest to
A) 0.3
B) 0.5
C) 0.68
D) 0.83

Unauthorized copying or reuse of any part of this page is illegal.


32
10 CONTINUE
1
Questions 33-42 are based on the following 35 24 sous. She burst into tears of a gratitude which I
passage. could perceive was unfeigned, because she was
unable to utter a word. She had probably never
This passage is adapted from a letter written by Thomas before received so great an aid.
Jefferson to James Madison. It was originally written in This little attendrissement1, with the solitude
1785, when Jefferson was residing in France. 40 of my walk led me into a train of reflections on
that unequal division of property which occasions
Seven o’clock, and retired to my fireside, I the numberless instances of wretchedness which I
have determined to enter into conversation with had observed in this country is absolutely
you; this [Fontainebleau] is a village of about concentered in a very few hands, having revenues
Line 5,000 inhabitants when the court is not here and 45 of from half a million guineas a year downwards.
5 20,000 when they are, occupying a valley thro’ These employ the flower of the country as
which runs a brook, and on each side of it a ridge servants, some of them having as many as 200
of small mountains most of which are naked rock. domestics, not laboring. They employ also a great
The king comes here in the fall always, to hunt. number of manufacturers, and tradesmen, and
His court attend him, as do also the foreign 50 lastly the class of laboring husbandmen2. But
10 diplomatic corps. But as this is not indispensably after all these comes the most numerous of all the
required, and my finances do not admit the classes, that is, the poor who cannot find work. I
expence of a continued residence here, I propose asked myself what could be the reason that so
to come occasionally to attend the king’s levees, many should be permitted to beg who are willing
returning again to Paris, distance 40 miles. 55 to work, in a country where there is a very
15 This being the first trip, I set out yesterday considerable proportion of uncultivated lands?
morning to take a view of the place. For this These lands are kept idle mostly for the sake of
purpose I shaped my course towards the highest game. It should seem then that it must be because
of the mountains in sight, to the top of which was of the enormous wealth of the proprietors which
about a league. As soon as I had got clear of the 60 places them above attention to the increase of
20 town I fell in with a poor woman walking at the their revenues by permitting these lands to be
same rate with myself and going the same course. labored.
Wishing to know the condition of the laboring I am conscious that an equal division of
poor I entered into conversation with her, which I property is impracticable. But the consequences
began by enquiries for the path which would lead 65 of this enormous inequality producing so much
25 me into the mountain: and thence proceeded to misery to the bulk of mankind, legislators cannot
enquiries into her vocation, condition and invent too many decides for subdividing property,
circumstance. She told me she was a day labourer, only taking care to let their subdivisions go hand
at 8. sous or 4 d. sterling the day; that she had in hand with the natural affections of the human
two children to maintain, and to pay a rent of 30 70 mind. The descent of property of every kind
30 livres for her house (which would consume the therefore to all the children, or to all the brothers
hire of 75 days), that often she could get no and sisters, or other relations in equal degree is a
employment, and of course was without bread. As politic measure, and a practicable one. Another
we had walked together near a mile and she had means of silently lessening the inequality of
so far served me as a guide, I gave her, on parting 75 property is to exempt all from taxation below a

Unauthorized copying or reuse of any part of this page is illegal.


33
11 CONTINUE
1
certain point, and to tax the higher portions of 33
property in geometrical progression as they rise. Jefferson’s central claim in the passage is that
Whenever there is in any country, uncultivated A) the current system of inheritance and
lands and unemployed poor, it is clear that the ownership is unlikely ever to change.
80 laws of property have been so far extended as to B) wealth should be redistributed in a way
violate natural right. The earth is given as a that benefits the majority of society.
common stock for man to labour and live on. If, C) the unemployed should use their energies
for the encouragement of industry we allow it to to work the land, rather than ask for money.
be appropriated, we must take care that other D) everybody has the same natural right to as
85 employment be furnished to those excluded from much property as he or she thinks
the appropriation… necessary.

1 emotion 33
2 farmers Jefferson uses the example of the woman he
meets on the road in order to
*sous, sterling, livres, guineas refer to units of currency. A) provide a moving anecdote that will
distract his reader from larger issues.
B) illustrate his surroundings with an atypical
example of the native people.
C) support the claim that the economic
situation is worse than Madison may think.
D) transition from his specific experience to a
more general argument.

35
Over the course of the passage, the main focus
shifts from
A) a description of the locale to a broader
discussion of principles.
B) a humorous anecdote to a position on a
popular U.S. debate.
C) an analysis of the king’s situation to an
analysis of an unemployed person’s.
D) Jefferson’s experiences in France to
Madison’s experiences in the U.S.

Unauthorized copying or reuse of any part of this page is illegal.


34
12 CONTINUE
1
36 40
The main purpose of the passage’s Which choice best describes Jefferson’s
information about Fontainebleau is to attitude towards socio-economic conditions
A) provide a picturesque counterpoint to the in France?
terrible poverty that Jefferson witnesses. A) He approves of the king’s policies on most,
B) describe a foreign land to someone who has but not all, issues.
never visited France. B) He sees France as an isolated case, with
C) contextualize the anecdote and argument unique conditions not applicable to other
that follow. countries.
D) interest the reader in the narrative before C) He is affronted at the few opportunities
changing topics. given to the poor.
D) He is pleased that the United States does
37 not experience the same conditions as
Jefferson implies that the initial purpose of France.
his walk was to
A) better understand the physical geography 41
around him. Jefferson’s statement that he is conscious that
B) get out of the town in order to think more “an equal division of property is
clearly. impracticable” (lines 63-64) implies that he
C) understand the condition of non-American A) has decided that there is no point in
laborers. pursuing his goals, because they would
D) avoid running into the king and his ultimately prove impossible to carry out.
courtiers. B) knows that his ideals are not entirely
realistic, yet still believes that some
38 improvement is possible.
Which choice provides the best evidence for C) does not care about what is realistically
the answer to the previous question? possible, but is only concerned with the
A) lines 10-14 (“But as ... miles”) way that things should be.
B) lines 15-16 (“This ... place”) D) believes that even though property cannot
C) lines 19-21 (“As soon ... course”) be divided equally, it should still be divided
D) lines 22-23 (“Wishing ... her”) according to a man’s abilities.

39 42
As used in line 58, “game” most nearly means Which choice provides the best evidence for
A) tricks to be played. the answer to the previous question?
B) play to be undertaken. A) lines 58-62 (“It should ... labored”)
C) sports to be practiced. B) lines 64-66 (“But the ... mankind”)
D) animals to be hunted. C) lines 66-70 (“legislators ... mind”)
D) lines 78-81 (“Whenever ... right”)

Unauthorized copying or reuse of any part of this page is illegal.


35
13 CONTINUE
1
Questions 43-52 are based on the following 35 words in a particular language. Thus, experience
passage and supplementary information. is necessary to become an expert in SCRABBLE.
And, in fact, SCRABBLE skill has been found to
This passage is adapted from David Z. Hambrick, “Winning correlate positively with the amount of time
SCRABBLE and the Nature of Expertise,” (c)2015 Scientific people spend engaging in SCRABBLE-related
American. 40 activities. In one study, using official SCRABBLE
ratings as an objective measure of skill,
SCRABBLE has been one of the most popular researchers found that groups of “elite” and
board games in the world for decades. And, now, “average” SCRABBLE players differed in the
as an increasingly popular domain for scientific amount of time they had devoted to things like
Line research on expertise, it is giving psychologists a 45 studying word lists, analyzing previous
5 better understanding of the underpinnings of SCRABBLE games, and anagramming—and not by
complex skill and a clearer picture of the origins a little. Overall, the elite group had spent an
of greatness. The overarching goal of this average of over 5,000 hours on SCRABBLE study,
research is to better understand the interplay compared to only about 1,300 hours for the
between “software” and “hardware” aspects of the 50 average group.
10 cognitive system. Software factors include Clearly, expert SCRABBLE players are to some
knowledge and skills that are acquired through degree “made.” But there is evidence that basic
experience, whereas hardware factors include cognitive abilities play a role, too. In a study
genetically-influenced abilities and capacities. recently published in Applied Cognitive
SCRABBLE is ideal for research on how these 55 Psychology, Michael Toma and his colleagues
15 factors interact not only because it is relatively found that elite SCRABBLE players outperformed
easy to find research participants from a wide college students from a highly selective university
range of skill, but because it can be imported into on tests of two cognitive abilities: working
the lab. memory and visuospatial reasoning. Working
The basic goal of SCRABBLE is to create 60 memory is the ability to hold in mind information
20 intersecting words by placing lettered tiles on a while using it to solve a problem, as when
board containing a 15 x 15 grid. Knowledge is, of iterating through possible moves in a SCRABBLE
course, critical for success in this task. If you want game. Visuospatial reasoning is the ability to
to become a great SCRABBLE player, first and visualize things and to detect patterns, as when
foremost, you have to know a lot of words. You 65 imagining how tiles on a SCRABBLE board would
25 also need to be adept at identifying potential intersect after a certain play. Both abilities are
plays. Finally, you have to know SCRABBLE influenced by genetic factors.
strategy—or what aficionados call “rack Further evidence pointing to a role of these
management”—such as how to keep a good mix of abilities in SCRABBLE expertise comes from a
consonants and vowels. 70 recent brain imaging study by Andrea Protzner
30 People aren’t born with this type of and her colleagues at the University of Calgary.
specialized knowledge. Research indicates that we Using functional magnetic resonance imaging
may come into the world equipped with the (fMRI), these researchers recorded the brain
building blocks for complex skills such as math, activity of SCRABBLE players and control subjects
but certainly nothing as specific as knowledge of 75 as they performed a task in which they were

Unauthorized copying or reuse of any part of this page is illegal.


36
14 CONTINUE
1
shown groups of letters and judged whether they Survey results of members with National Scrabble
formed words. (fMRI measures brain activity by Association Ratings versus college students at a liberal
detecting changes in blood flow within different arts college who have played Scrabble at some point but
regions of the brain.) The major finding of this have never competed.
80 study was that competitive SCRABBLE players
recruited brain regions associated with working Table 1.
memory and visual perception to perform this Variables Scrabble experts College students
task to a greater degree than the control subjects
Number of people 114 147
did.
Average days per year
85 What might explain SCRABBLE experts’ 221.37 8.1
spent playing scrabble
superiority in working memory and visuospatial
reasoning? For the same basic reason that Table 2.
basketball players tend to be tall, a likely Scrabble experts College students
Questions who answered who answered
explanation is that people high in working yes yes
90 memory and visuospatial reasoning abilities are
Have you ever studied
people who tend to get into, and persist at, words from the Official
playing SCRABBLE: because it gives them an Scrabble Player’s 99% 5%
Dictionary or a similar
advantage in the game. This explanation fits with
source?
what behavioral geneticists call gene-
Are there words that you
95 environment correlation, which is the idea that
know are legal in
our genetic makeup influences our experiences. Scrabble, but you do not 89% 26%
These findings add to an emerging know what the word
means?
understanding of complex skill that may
ultimately bring expertise within reach of a larger When you play Scrabble
do you always keep track
100 number of people than is currently the case. of the letters that have
been played, so you know 80% 4.8%
if you are likely to get a
* Changed for cultural accuracy.
rare letter that you might
need?

When your opponent has


the Scrabble board facing
him or her, do you
sometimes mentally
17% 38%
rotate the board to
imagine what it would
look like if it were facing
you?

Do you know the point


value for every square on
the board (e.g. double or 70% 3%
triple word/letter score
tiles?)

Unauthorized copying or reuse of any part of this page is illegal.


37
15 CONTINUE
1
43
The passage makes the most extensive use of
which kind of evidence?
A) expert testimony.
B) scientific studies.
C) secondary sources.
D) statistical analysis.

44
The main effect of the term “hardware” in line
9 is to suggest that the human brain
A) is more like a computer than people realize.
B) has particular characteristics that cannot
be changed.
C) can be essentially reprogrammed through
people’s actions.
D) can be improved through the use of
computers.

45
Evidence from the passage suggests that elite
SCRABBLE players would be most likely to
outperform non-elite SCRABBLE players in
which of the following?
A) Gaining admission to a selective university.
B) Playing a board game like chess.
C) Solving a mathematical equation.
D) Learning a foreign language.

46
Which choice provides the best evidence to
refute the claim that two people performing
the same activity would utilize their brain
capacities to a similar extent?
A) lines 21-24 (“Knowledge ... words”)
B) lines 40-47 (“In one ... little”)
C) lines 77-79 (“fMRI ... brain”)
D) lines 79-84 (“The major ... did”)

Unauthorized copying or reuse of any part of this page is illegal.


38
16 CONTINUE
1
47 51
As used in line 41, “objective” most nearly Based on the passage and the chart, college
means students playing SCRABBLE are more likely
A) perceptible. than elite players to
B) sensory. A) make an effort to visualize the board from
C) intentional. a different perspective, because they are
D) unbiased. less able to rely on visuospatial reasoning.
B) have difficulty remembering which letters
48 have been played, because they use their
The main purpose of paragraph 3 (lines 30-50) working memory less.
is to C) forget to study, because they are less
A) trace the non-cognitive factors associated invested in winning any particular game.
with SCRABBLE expertise. D) spend time learning words, because they
B) show that average SCRABBLE players lack have less inherent talent than elite players.
the genetic advantages of elite players.
C) examine the value of childhood training in 52
predicting SCRABBLE success. According to the chart, elite SCRABBLE
D) demonstrate that hard work is all it takes players are most likely to do which of the
to become an elite SCRABBLE player. following?
A) Memorize words to play in SCRABBLE.
49 B) Keep track of letters played by the
The author implies that people who opponent.
experience great success in a given activity are C) Memorize point values allotted to each
often born with letter.
A) advantages uniquely suited to that activity. D) Imagine the board from a different point of
B) a drive to succeed in any activity. view.
C) parents who value success in that activity.
D) a desire to participate in many activities.

50
Which choice provides the best evidence for
the answer to the previous question?
A) lines 35-40 (“Thus ... activities”)
B) lines 47-50 (“Overall ... group”)
C) lines 63-66 (“Visuospatial ... play”)
D) lines 87-93 (“For ... game”)

STOP
If you finish before time is called, you may check your work on this section only.
Do not turn to any other section.

Unauthorized copying or reuse of any part of this page is illegal. 39


17
40
Test 1
Reader’s Guide

44
41
< Passage 1 >

Summary
Line
1-27: The narrator and his companions enter a chamber. The narrator notices that
one of his companions, Ruth, had on him a gaze of dread and fear while
looking at a patterned circle in the floor.
28-49: The narrator notices that the pattern in the ring is unusually symmetrical and
feels dread because in the middle of the ring is a miniature replica of the
giant track in the poppied valley.
50-64: The narrator picks up one of the pyramid figures in the ring and then notices
that the ring starts to move and becomes animated.
65-92: The figures revolve, raise themselves, erect, and finally melt into a word.
93-109: The figures then form a pedestal from which springs a star of five arms. A
troll sweeps toward the narrator.

Tone

Dreadful and tense

Main Idea

The narrator narrates a story of when they encounter a hostile figure in a chamber.

Vocabulary
preservation: 보존; the action of maintaining

flagging: 쇠약해가는; becoming weaker

cavern: 커다란 동굴; a massive cave


crevice: 바위에 생긴 틈; a crack forming an opening

primrose: 프림로스 (꽃 종류); name of a flower

conventionalize: 양식화하다; to represent in an ordinary matter

platinum: 백금; name of a metal

lustrous: 윤기가 흐르는; shining, brilliant

ferment: 발효시키다, 불러일으키다; to act as a ferment, to cause

grotesque: 터무니없는; bizarre odd

42
56
pedestal: 받침대; a supporting piece, base

poise: 침착, 균형; composure, equilibrium

57
43
< Passage 2 >

Summary
Line
1-3: Undesired side effects of adverse road conditions.
4-16: Traffic congestion affects many aspects of socioeconomic life.
17-21: Cause of traffic jams – simple and complex.
23-34: Why forecasting and traffic jams are a hard problem. “Fundamental diagrams
of traffic” and relation between density and traffic flow + velocity.
35-46: How a global ledger where future participants volunteer their future itineraries
would help forecast. However, this ledger is highly unrealistic while Google
search data could work.
47-69: Google search data reveals the individuals’ state or intent.
70-88: Proliferation of advanced mobile applications diminish the effectiveness of
this method. But, there is still no method of knowing traffic jams hours
ahead.

Main Idea

Traffic congestion affects many aspects of socioeconomic life and there are a mix of
simple and complex causes to this issue. Forecasting traffic jams is a hard issue
because the relation between passenger density vs. traffic flow + velocity is not
always predictable. A global ledger where future participants volunteer their future
itineraries would be the best way to predict traffic flow. Unfortunately, this is
highly unrealistic and there is still no way of knowing traffic jam hours ahead.

Tone

Informative

Vocabulary
adverse: 부정적인, 불리한; negative, unfavorable

congestion: 혼잡, 막힘; overcrowding

forecast: 예측하다, 예측; to predict, a prediction

empirical: 경험에 의거한; based on experience

intuition: 직감; direct perception of truth

itinerary: 여행 일정표; a detailed plan for a journey

44
58
algorithm: 알고리즘; a set of rules for solving a problem in steps

queue: 대기행렬, 줄을 서서 기다리다; a line, to wait in lines


dissipate: 소멸하다, 허투루 쓰다; to disperse, to deplete

proliferation: 급증, 확산; a rapid spread or increase

crowdsource: 대중의 참여로 인해 얻은 정보를 사용하다; to utilize information obtained


by the general public (often through the internet)

59
45
< Passage 3 >

Summary
Line
1-20: The ivory-billed woodpecker is an elusive species that is difficult to document
due to a multitude of reasons.
24-42: Climate change affects BBD infestation varies due to the huge geographical
habitation of beech trees.
43-57: Drought decreases nitrogen uptake of beech trees, which prevents them from
host invasion.
58-80: Introducing BBD-resistant beech trees north will help speed migration
according to the drought conditions.
81-95: Drought and elevated temperatures, along with BBD-infestation greatly
diminish the population of beech trees.

Main Idea

Both climate change and elevated temperatures are contributing to the diminishing
population of BBD, but alternative solutions are cropping up to address the issue.

Tone

Informative

Vocabulary
additional: 추가의; added or extra
persistence: 고집; sticking to something, not giving up
rarity: 희귀성; something that is very infrequesnt or scarce
wariness: 신중, 경계심; caution, watchfulness
vantage: 우세, 유리; position giving an advantage
feasibility: 실행 가능성; possibility
adhere: 들러붙다; stick to
foraging: 수렵 채집; to search for food or provisions
vegetation: 초목; plant life
application: 지원, 응용; use, the act of putting to a special use or purpose

46
60
< Passage 4 >

Passage 1

Summary
Line
1-30: There are still members in the office that continuously strive to exploit and
gain power for their own good
31-41: There are probably people in the office that don’t deserve their own position,
but just have the job to do a lot of work.

Main Idea

There are and will always be men in office that continuously work to gain power for
their own benefit rather than for the country or company. There are probably people
that are employed simply to do a lot of work, rather than actually be effective and
attribute towards the benefit of the country.

Tone

Criticizing

Passage 2

Summary
Line
42-48: If power of removal were to be unlimited and without any bias, the
government should be considered as the role example.
49-70: However, the Senators claim that this process requires a competitive
examination, even though there are probably people in the office that are
unfit for the job
71-83: If there are any lower class jobs open, Democrats will come fill them up,
while Republican employees will be able to apply and get the higher ranked
jobs.
84-90: The Republicans are the ones who mostly get what they want, and therefore,
the government is not necessarily the best measure.

47
61
Main Idea

The government is corrupt in a way that the Democrats are usually the ones who
receive “lower” class jobs and the Republicans are given more opportunities to work at
higher jobs. The congress denies it, but the office is controlled over the abuse of
power.

Tone

Criticizing

Vocabulary
doctrine: 교리, 신조, 정책; body of beliefs or ideas
reform: 개혁하다; to change for the better
degradation: 비하; humiliation, deprivation
bureau: 책상, 사무실; department, drawer
inefficient: 비효율적인; achieving a goal in a more costly manner than necessary
whim: 기분, 변덕; a sudden and impulsive action
delusion: 망상, 착각; a false belief or opinion, hallucination
mockery: 조롱, 엉터리; ridicule, act of making fun of someone or something
executive: 경영진, 행정부; branch of government that enforces the laws
clerkships: 서기나 목사의 직; the job of clerk
vacant: 비어있는; empty
incumbents: 재임자, 필요한; required, one who holds a specific office at the time
spoken of
paltry: 보잘것없는, 시시한; trifling, insignificant
Democrats: 민주주의자
Republicans: 공화주의자

62
48
< Passage 5 >

Summary
Line
1-10: The origin and definition of the term “dietary fiber.”
11-23: Denis Burkitt re-popularized the idea of dietary fiber and since then it has
been used in many ways for beneficial health effects.
24-39: Prebiotics original and refined definition.
40-49: The differences between prebiotics and dietary fiber.
50-61: Historical consumption of prebiotics and current consumption of prebiotics
62-75: An important mechanism of action for dieatry fiber and prebiotics is
fermentation.
76-95: The benefits and effects of dietary fiber beyond improved laxation

Main Idea

The progressive terms, research and consumption of dietary fiber and prebiotcs. Its
new found benefits on physical and mental health should be recognized and promoted
more.

Tone

Informative

Vocabulary
dietary: 음식물의; related to the food someone eats
fiber: 섬유; a tough complex carbohydrate that the body cannot digest
laxative: 완화제; loosens and promotes normal bowel elimination
obesity: 비만; overweight
cardiovascular: 심혈관의; of the heart and vessels
cohort: 집단, 지지자; a group of people from a given time period
microorganism: 미생물; a small living plant or animal seen only with a microscope
composition: 구성, 작성; the make up of something
gastric: 위의; pertaining to the stomach
acidity: 산성; the strength of an acid
intestinal: 장의; pertaining to the intestines
colonized: 식민지로 만들다; to make settlements

49
63
solubility: 용해도; the ability of one substance to dissolve in another at a given
temperature and pressure
fermentability: 발효가능성
viscosity: 점도; a liquid’s resistance to flowing
determinants: 결정 요인; cause

64
50
Test 2
Reader’s Guide

53
51
  


 


               !!


#
"       !      !$

'%&       !! $    ! $   (    
           $ !  )  
%"&%     !   * $$ )     !+  
&&'
    , +  )     )       
.-!        -( )      
'%'#         ( $!     
'/0       -   $  ! .        

#1/%     !  $ !    ( ) $  
/&0
     !2     .   )     $ (    
 $!    $

 3

  )!+     $! $        -(  !
$  +  $      $! --!(    $

4

 

5)!

 768    $ -!!   +)!   -      - $ 
     ! $ 
 - 9:;( <=;8  * 

52
  >?8   ) $  +  )  )  +!   )  !!
   -+! !
) @A7B8  ) $ !+ +  $$!
   CD; GEFE IH8J  )  (  (  *!
-)!JLK MN;8J)!     !   ! !
$$! POPO RQS8  $!- 

53
   %

 

 



0     4 + T  .-    !(  - +! -!!
%1%/    !.   + ) -!!   )$  +  ) 
.-  !! ) -!!    
%0&/    U   - $  !.    - $  ! !  $ !. ( 
  !  !! +!   +  
&0"&    V !!  ) )!       !.     !
- $  ! 

  %

 

"""#    W     !.    ! !!
"/'0     $  ! ( !  !.     $ ! !! 
1"   ,   $     $   + !.   $  ) 
!! 
'#%   ! !!    $$!   )   ! +!+  
!!  !+ !! +    !!-! - 
!  $  $ 
#&/1   4 ! V     !!   !.   !!

 3

  !.   ) -!!    (    $  ! ( ! 
!.   $   ! !! 

54
4

3$ +

5)!

!!  XYIH8   -    )  $ +(  ( (
    -
  [Z\S8 !( !
   ]78 + 
 _^`8  )!
!! ba\S8      !    -
!!-! dceB8 $( ! (  - !  +!
! fhg iS8         !  -  

55
   &

 


&0        ! +    - + $$   !( )  j $
!   lk     . 

#&"     ! -!  )  !!  ! +


&'"&   V!   +  -   -!   - ! !   
-! (    -!   T     -! 
"""   W! +       - ! + 
  -! 
' /
    W! +    )$ ! T !$   ( )  $$  
  -   !  ! 

 3

m!   lk     - ! +  (    ) !-$!
  -!  ! +  (  !  )$ ! T !$  

4

 +(  +

5)!

+ ! no p@B8 $( ! (  +!+    )! + !
JqrB8       $  !  
 s<;8 ) +  )! ! 
-+  [t( vu8    -  $ -+
$! _x\w \S8    
! J{zy8J  -     +!-   !

56
   "

 



"    J4    !!(         $!!  

'&/   |$$       !        $ (    
  $  -!  +
&0%  4  ! |$$    $  !    ) $!
     !
&/   V! |$$  !      - )!   ) ! *!!( 
  !  )!+    ! ! -  - $  ) . 
      !  ! --! -  .

 3

} --! ! -  . (  - --! !  +  - . 
)!   -

4

U -( }$!+

5)!

) ~8J  !! 


 - )! €( ƒ‚8  )j  )     !
* „X( 6B8  *  $ $ 
 …X8   $ ) $!( $!
  †‡ˆ8   $ ) -! $$!( j(     
       )  
!+ ‰‹Š[ Œ`8  !!
 -)! Ž MN;8 -)! $ ) -$    -!  ) 
         -!    
)+  ’‘“ ”8  ) - $  ! !

57
   '

 



/   ))!   !-       .-   
$!  +  

0%0   4 )    ))! -!(      ! $   
    !  !!
&1'1   -!   )   )!  -! ))!(  •!– ))!
  -!  +            + -!
'
#  V! •!– ))! -!    $  .-(  $  -!
   !   !!
//"    $ +!      +  -!   -!
  !  ))!       -+ -!   ! 
 -
/'0  3   !!  --! )  )     +  -!
   !!  $      +  - -! 
  
0#
11  4  $   !-  --!  .-    $

 3

U.-    ) $!  +   (      + 
))! -!

4

—  +( 3$ +

5)!

.-  ˜6s Q™8  !! $  .-

58
+ šœ›S ž;8     ) !! ! 

$ Ÿ¡ 8  -   ! (   )(  --    !! 

    $+! -     +

-$  ‹¤N¢¥£ ¦§ NM\S8  -$  ) 

+  -! u¨ ©¥ @;8 $( ! (  )  -    +!+
     + !  -! 

--Jǻ8   $  !   $ +   - !  

59
60
SAT Writing

61
61
SAT Writing
Concepts & Practice Questions

62
62
Z9 ÆñèÜàî ¸âîààèàéð

Z.Z9 ÌñÜåàÝð ÏàîÜ ¸âîààèàéð ¼îîëîï õºãàÝæ äá ïñÜåàÝð èÛðÝãàï ÝëîîàïìëéÞäéâ ÏàîÜ

ÍÛÜçà Z9Z õìâ9 Z|

Plurality of subject Verb


Singular is, was, has, talks, misses
are, were, have, talk, miss
Plural

¸9 ½äîïð âàð îäÞ ëá ðãà èëÞäáäàîï õìîàìëïäðäëé# ÞëñÜçà ÝëèèÛ# ÛÞå ÛéÞ ÛÞò

Z ÈîàìëïäðäëéÛç ìãîÛïà

àô Íãà Þëâï äé ðãà ÝÛâà óÛï çÛîâà9 õÑ

ÍÛÜçà Z9e Íöìàï ëá ìîàìëïäðäëéï õìâ9 Z|

õÌðñÞàéðï ïãëñçÞ Üà ÛÜçà ðë ðàçç ðãà ÞäááàîàéÝà Ûèëéâ ìîàìëïäðäëéï# ÝëéåñéÝðäëéï# ÛéÞ

ÝëéåñéÝðäòà ÛÞòàîÜï

Prepositions of Examples
Place on, beneath, over, under, above
Time at, in, on, by, for, during
into, out of, across, along, through, from,
Direction
to
Other prepositions like, despite, in spite of

e ºëèèÛ ÝëèèÛ ìãîÛïàï Üàðóààé ïñÜåàÝð ÛéÞ òàîÜ

àô Íãà ðàÛè ëá ÛÞòäïëîï# Ûîîäòäéâ ÛãàÛÞ ëá ïÝãàÞñçà# óàîà èàð Ûð ðãà Ûäîìëîð Üö ðãà ðàÛè

çàÛÞàî9 õÑ

q ¸ÞåàÝðäòàï ÛéÞ ÛÞòàîÜï

ä9 ÐãÛð äï Ûé ÛÞåàÝðäòච. Áð ÞàïÝîäÜàï éëñéï9

àô ÐÛðÝãäéâ âÛèàï èÛæà öëñ ãÛììö9 õÑ

ÍÛÜçà Z9q Íöìàï ëá ÛÞåàÝðäòàï õìâ9 Z‡

ÇîäâäéÛç ÛÞåàÝðäòà ìîàððö# ãÛéÞïëèà# æäéÞ

ÏàîÜ  äéâ Û áçöäéâ ÜäîÞ

Íãàîà Ûîà ðóë âñÛîÞï ìëïðàÞ Ûð ðãà Þëëî9 õÛÞå Ç Á


ÏàîÜ  àÞ
ìëïðàÞ ãäè Ûð ðãà Þëëî õÛÞå Ñ

ðë  ÏàîÜ Á ãÛòà èÛéö áîäàéÞï ðë ãàçì èà9

óãëõè# óãäÝã# óãëïà 


Á çäæà Û ÝãÛäî óãäÝã ãÛï áëñî çàâï9

äéÝëèìçàðà

ü ¼ôÝàìðäëé Ëñçà ° ÏàîÜäéâ ´ ÌäéâñçÛî éëñé

63

63
ää9 ÏàîÜäéâ ´ Æëñé

äää9 Üà  ÏàîÜäéâ ´ ÏàîÜ

àôÍãà Üëôàï ìñïãàÞ áîëè ðãà ðÛÜçà äï àèìðö9 õÑ

ÍÛÜçà Z9| Àëó ðë äÞàéðäáö óãàðãàî ðãà òàîÜ îàçÛðàÞ óëîÞ ÝÛé ÛÝð Ûï Û òàîÜ

ºÛééëð ÛÝð Ûï Û Ï ºÛé ÛÝð Ûï Û Ï

Ï  äéâ âëäéâ äï âëäéâ


Üà  Ï  äéâ

ìÛïð ìÛîðäÝäìçà âëéà äï âëéà


Üà  ì9ì

ðë  Ï ðë âë ãÛï âëéà
ãÛòà  ì9ì

äò9 ÐãÛð äï Ûé ÛÞòàîܶ . Áð ÞàïÝîäÜàï Ûçç ìÛîðï ëá ïìààÝã àôÝàìð éëñéï

ÍÛÜçà Z9‡ Íöìàï ëá ¸ÞòàîÜï õìâ9 Z‡

Íöìàï ¼ôÛèìçàï

ûçö óëîÞï ÝÛîàáñççö# Ýçëïàçö

ðë  ÏàîÜ Á óàéð ÛÜîëÛÞ ðë ïðñÞö ¼éâçäïã

¸ÞòàîÜ ÝçÛñïà Ðãàé Á óÛï öëñéâ# Á ñïàÞ ðë û

ÈîÛÝðäÝà Êñàïðäëéï

Z9 ¹îÛòàãàÛîð ÛéÞ Íãà ÈÛðîäëð# ðóë èëòäàï óãäÝã ïðÛî Åàç ¿äÜïëé# ÝÛìäðÛçä÷àï ëé ðãà ÞîÛèÛðäÝ

ìëðàéðäÛç ëá ðãàäî ãäïðëîäÝÛç ïàððäéâï9

¸ ÝÛìäðÛçä÷àï

¹ ÝÛìäðÛçä÷à

º óÛï ÝÛìäðÛçä÷äéâ

» ãÛï ÝÛìäðÛçä÷àÞ

e9 ¸é äÝà ÝîàÛè ïñéÞÛà# àïìàÝäÛççö óãàé ðëììàÞ óäðã ðãîàà ëî áëñî Ýãàîîäàï# èÛæà Û ÞàçäÝäëñï Þàïïàîð9

¸ èÛæà

¹ èÛæàï

º óàîà èÛæäéâ

» Ûîà èÛæäéâ

q9 ¸çàôÛéÞîà »ñèÛïêï éëòàç# Íãà Íãîàà Åñïæàðààîï# ðàçç Û ÞîÛèÛðäÝ ìëçäðäÝÛç ÛéÞ ÛÞòàéðñîà ïðëîö ÛÜëñð

ðãà îëöÛç ½îàéÝã èäçäðÛîö9

¸ ðàçç

¹ Ûîà ðàççäéâ

º óàîà ðàççäéâ

» ðàççï

|9 ¼ÞóÛîÞ ðîäàÞ ðë áëÝñï ëé óãÛð ðãà ìëçäÝà ëááäÝàî óÛï ïÛöäéâ# Üñð Ûçç ðãà ðãäéâï ãà ïðäçç éààÞàÞ ðë Þë

Üàáëîà ðãà äéðàîòäàó+ÛéÞ íñäÝæçö# ïë ðãÛð ãà ÝëñçÞ Üà ëé ðäèà+óÛï îÛÝäéâ ðãîëñâã ãäï èäéÞ9

¸ óÛï

¹ óàîà

º ãÛï Üààé

» äï

64

64
SV/Noun Agreement

1.
Braveheart and The Patriot, two movies which star Mel Gibson, capitalizes on the dramatic
potential of their historical settings.
A) NO CHANGE
B) capitalize
C) was capitalizing
D) has capitalized

2.
An ice cream sundae, especially when topped with three or four cherries, make a delicious
dessert.
A) NO CHANGE
B) makes
C) were making
D) are making

3.
Alexandre Dumas’s novel, The Three Musketeers, tell a dramatic political and adventure story
about the royal French military.
A) NO CHANGE
B) are telling
C) were telling
D) tells

4.
Edward tried to focus on what the police officer was saying, but all the things he still needed to
do before the interview—and quickly, so that he could be on time—was racing through his mind.
A) NO CHANGE
B) were
C) has been
D) is

65
5.
Using a flock of Doves—which, in this case, are tiny satellites made by Planet Labs—are one
way to monitor the real-time effects of climate change.
A) NO CHANGE
B) is
C) were
D) have been

6.
Gabriella wanted to be a doctor until she learned how many tests before, during, and after
medical school is required to graduate.
A) NO CHANGE
B) was
C) are
D) has been

7.
The calendar and the wheel, each an example of Mesopotamian ingenuity, are among the
inventions we often take for granted today.
A) NO CHANGE
B) was
C) is
D) has been

8.
Reading fiction, cooking from scratch, and spending time in his garden is some of the ways
Frank reminds himself to relax.
A) NO CHANGE
B) are
C) has been
D) was

66
9.
Science, law, and computer programming are all a profession suited for left-brained thinkers.
A) NO CHANGE
B) a career
C) an area of specialization
D) fields

10. A number of fish that was caught included endangered species.


A) NO CHANGE
B) that were
C) that has been
D) that is

67
e9 Èîëéëñé ¼îîëîï

e.Z9 ÆñèÜàî ¸âîààèàéð õÌãàDÀàDÁðDÍãàö

¸9 Íãà ìîëéëñé ðãà ïñÜåàÝð îàáàîï ðë èñïð Üà ïäéâñçÛî äá ðãà ïñÜåàÝð äï ïäéâñçÛî ÛéÞ òäÝà

òàîïÛ9 àô Èàëìçà óÛðÝãàÞ ðãà óãäðà ãëñïà Ûï äð óÛçæàÞ Üö9 õÑ

ÍÛÜçà e9Z

Èàëìçà Íãäéâï

ÌäéâñçÛî ãàDïãà äð

ÈçñîÛç ðãàö ðãàö

e.e9 Èîëéëñé ºÛïà õÁDÅà

¸9 ÌñÜåàÝðäòà Èîëéëñéï° ÎïàÞ Ûï ðãà ïñÜåàÝð ëá Û òàîÜ9

¹9 ÇÜåàÝðäòà Èîëéëñéï° ¸é ëÜåàÝð äï Û éëñé õëî ìîëéëñé ðãÛð äï âëòàîéàÞ Üö Û òàîÜ ëî Û

ìîàìëïäðäëé9

àô Åà óàéð ðë ïÝãëëç9 õÑ

ÌñÜåàÝðäòà Èîëéëñé ÇÜåàÝðäòà Èîëéëñé Èëïïàïïäòà ¸ÞåàÝðäòà Èëïïàïïäòà Èîëéëñé

Á D Ðà Åà D Îï Åö D Çñî Åäéà D Çñîï

Òëñ Òëñ Òëñî Òëñîï

Àà D Ìãà D Íãàö Àäè D Ààî D Íãàè Àäï D Ààî D Íãàäî Àäï D Ààîï D Íãàäîï

Áð D Íãàö Áð D Íãàè Áðï D Íãàäî Áðï D Íãàäîï

àô Z Á áëñéÞ ãà ïðñìäÞ9 õÑ

àô e ¼òàîöëéà àôÝàìð Á óäçç ÛððàéÞ9 õÑ

àô q Íãà ìñîìçà âäáð Üëô äï öëñîï# Üñð ðãà öàççëó ëéà äï èäéà ìîàïàéð9 õÑ

ü ¼ôìçÛéÛðäëéï ÛéÞ àôÛèìçàï

Z9 Äëëæ Ûð ðãà ÏàîÜ

Û9 Èîëéëñé Üàáëîà ÏàîÜ ´ ÌñÜåàÝðäòà Èîëéëñé

Ü9 Èîëéëñé Ûáðàî ÏàîÜ ´ ÇÜåàÝðäòà Èîëéëñé

àô Íãà åñÞâà áëñéÞ ïãà âñäçðö9 õÑ

e9 Äëëæ Ûð ðãà Èîàìëïäðäëé

Û9 ¸çóÛöï ÇÜåàÝðäòà Ûáðàî Èîàìëïäðäëé

àô ºÛé öëñ Þë ðãäï áëî Á¶ õÑ

q9 Íîö ëèäððäéâ Æëñé ÛéÞ

Û9 »àçàðà Æëñé ÛéÞ ÛéÞ äÞàéðäáö ìîëéëñé ÝÛïà

àô Íëè ÛéÞ èà óàéð ðë ïÝãëëç9 õÑ

e.q9 Èîëéëñé Ìãäáð õëéà.ëéà D öëñ.öëñ

¸9 Ðãàé öëñ ïðÛîð ðãà ïàéðàéÝà óäðã Û ìÛîðäÝñçÛî ìîëéëñé# Þë éëð ïãäáð ðë Ûéëðãàî ìîëéëñé Üàáëîà

ðãà àéÞ ëá ðãà ïàéðàéÝà9

àô Áá ëéà óÛéðï ðë èàèëîä÷à òëÝÛÜñçÛîö# öëñ ïãëñçÞ èàèëîä÷à îëëð óëîÞï9 õô

66
68
e.|9 ¸èÜäâñëñï Èîëéëñéï

¸9 Æë ÛéðàÝàÞàéð õÈîëéëñé ðãÛð äï éëð èàéðäëéàÞ ÜàáëîàãÛéÞ ÝÛééëð Üà ñïàÞ9

.
àô Á çäæà ÅàôäÝÛé Ýñçðñîà ÜàÝÛñïà Á ãÛòà Üààé ðãàîà õÑ

¹9 ºçàÛîàî õ¸çóÛöï ñïà ÝçàÛîàî Ûéïóàî äé ¸èÜäâñëñï ìîëéëñé íñàïðäëéï9

àô ËàÛÞäéâ äèìîëòàï ïðñÞàéðïê ïæäççï9 Áð äèìîëòàï ðãàäî èäéÞï9 õÑ

àô ËàÛÞäéâ äèìîëòàï ïðñÞàéðïê ïæäççï9 Áð äèìîëòàï ðãà ïðñÞàéðïê èäéÞ õÇ

e.‡9 ËàçÛðäòà Èîëéëñé õÐãëDÐãëè

¸9 Èàîïëé ÏàîÜ ëÜåàÝð .µ ºãëëïà óãë

¹9 Èàîïëé ÌñÜåàÝð òàîÜ .µ ºãëëïà óãëè

àô Àà ïÛó ÅÛîö óãë ãà çëòàÞ9 õÑ

A. ÌñÜåàÝð ÏàîÜ ÇÜåàÝð# óãäÝãDóãë  èëÞäáäàî° èëÞäáäàî ÝÛééëð èëÞäáö ÌñÜåàÝð Ûçëéà9

àô Á çäæà ðãà óîäðäéâï ëá Íëè ÛéÞ Âëãé# óãë ÝëéðäéñàÞ ðãà ÞäïÝñïïäëé9

àô ÄÛñîÛ# óãäÝã áÛòëîäðà ðãäéâ ðë Þë äï àÛðäéâ äÝà.ÝîàÛè# ãÛÞ Û ïàòàîà ðëëðãÛÝãà9 õÑ

ÍÛÜçà e9Z Íöìàï ëá ËàçÛðäòà Èîëéëñéï õìâ9 q’

Èàîïëé ÈçÛÝà Íäèà Íãäéâ

ÌñÜåàÝðäòà Ðãë Ðãàîà Ðãàé ÐãäÝã D ÍãÛð

ÇÜåàÝðäòà Ðãëè Ðãàîà Ðãàé ÐãäÝã D ÍãÛð

Èëïïàïïäòà Ðãëïà Çá óãäÝã D Ðãëïà D Áé óãäÝã

ÈîÛÝðäÝà Êñàïðäëéï

Z9 ÐàðçÛéÞ ÛîàÛï Ûîà Þäòàîïà àÝëïöïðàèï ðãÛð Ûîà ïëèàðäèàï ÞîÛäéàÞ áëî îàÛç àïðÛðà Þàòàçëìèàéð ëî

áçëëÞàÞ áëî ñïà Ûï îàÝîàÛðäëéÛç çÛæàï# óãäÝã äï Þäïðîàïïäéâ ÜàÝÛñïà Üëðã ÛÝðäëéï ìîàòàéð óàðçÛéÞï áîëè

ìàîáëîèäéâ ðãàäî ÝîäðäÝÛç# éÛðñîÛç ðÛïæï ëá áçëëÞ Ýëéðîëç ÛéÞ ðãà áäçðàîäéâ ëá óÛðàî ìëççñðäëé9

¸ çÛæàï# óãäÝã äï Þäïðîàïïäéâ ÜàÝÛñïà Üëðã ÛÝðäëéï ìîàòàéð

¹ çÛæàï# óãäÝã äï Þäïðîàïïäéâ ÜàÝÛñïà äð ìîàòàéðï

º çÛæàï# óãäÝã äï Þäïðîàïïäéâ ÜàÝÛñïà ðãàö ìîàòàéð

» çÛæàï9 Íãäï äï Þäïðîàïïäéâ ÜàÝÛñïà äð ìîàòàéðï

e9 ÄÛó ïðñÞàéðï Þë éëð ÛçóÛöï Ûììçö ðãàäî Þàâîààï ðë ðãà ìîÛÝðäÝà ëá çÛó± äéÞààÞ# áäàçÞï çäæà èàÞäÛðäëé

ÛéÞ ÜÛéæäéâ ÛéÞ áäéÛéÝà ïðäçç ãäâãçö òÛçñà ðãàè9

¸ ðãàè9

¹ ðãàïà îàïëñîÝàï9

º äðï9

» ðãàïà ÝîàÞàéðäÛçï9

q9 ¸çðãëñâã îëÜëðï óäçç ëéà ÞÛö ãàçì óäðã ãëñïàãëçÞ Ýãëîàï# äð óäçç éàòàî Ýëèìçàðàçö îàìçÛÝà èÛäÞï9

¸ äð

¹ ðãàö

º ãà

» öëñ

67
69
Pronoun Errors

1.
Law students do not always apply their degrees to the practice of law; indeed, fields like
mediation and banking and finance still highly value them.
A) NO CHANGE
B) these resources.
C) its.
D) these credentials.

2.
Although robots will one day help with household chores, it will never completely replace maids.
A) NO CHANGE
B) they
C) he
D) you

3.
One should obtain all necessary malaria shots before you visit Africa.
A) NO CHANGE
B) them visit
C) one visits
D) it visits

4.
As the children sang along to the old songs of The Sound of Music, they began to feel happy.
A) NO CHANGE
B) he
C) you
D) it

70
5.
For a boat to move forward, you must have enough force to push its own weight in water out of
the way.
A) NO CHANGE
B) it
C) we
D) they

6.
An Actor Prepares, Building a Character, and Creating a Role describe Constantin
Stanislavski’s system of acting; some are among the most read books on the craft.
A) NO CHANGE
B) it is
C) they are
D) ours are

7.
Though Dorothy’s shoes were silver in The Wizard of Oz, it was changed to red in the film.
A) NO CHANGE
B) they were
C) you were
D) she was

8.
Happiness is a state of mind we all strive to achieve; however, it is sometimes difficult to obtain.
A) NO CHANGE
B) he is
C) you are
D) they are

71
9.
When tourists visit the Las Vegas Strip, you will find Paris, Venice, pirates and a volcano—all
within a four-mile stretch!
A) NO CHANGE
B) yours
C) we
D) they

72
q9 ÈÛîÛççàçäïè ¼îîëîï

q.Z9 Ǹǹ

A. ¹ëðã ¸ ÛéÞ ¹# ¼äðãàî ¸ ëî ¹# éëð ëéçö ¸ Üñð Ûçïë ¹° Îïà ðãà ïÛèà ìîàìëïäðäëé áëî ¸ ÛéÞ ¹ äá

ìîàìëïäðäëé äï éàÝàïïÛîö9

àô Á âÛòà èö Üëëæï Üëðã ðë Íëè ÛéÞ Âëãé9 õÑ

q.e9 Ìàîäàï

A. Îïà ðãà ïÛèà ìîàìëïäðäëé áëî Û ïàîäàï ëá ÏàîÜï ëî Æëñéï9

àô Òëñ ÝÛé îàâäïðàî Üö èÛäç# áÛô# ëî Üö ìãëéà9 õÑ

ÈîÛÝðäÝà Êñàïðäëéï

Z9 ½ëîàéïäÝ Ûéðãîëìëçëâäïðï îàÝëòàî îàèÛäéï# ãÛòà àôÛèäéàÞ ìãöïäÝÛç ÝãÛîÛÝðàîäïðäÝï ðë Þàðàîèäéà

âàëâîÛìãäÝÛç ëîäâäéï# ÛéÞ çëëæ áëî àòäÞàéÝà ëá ðîÛñèÛ9

¸ ãÛòà àôÛèäéàÞ

¹ àôÛèäéà

º àôÛèäéàÞ

» Ûîà àôÛèäéäéâ

e9 ËàâÛîÞçàïï ëá Û ïðñÞàéðêï èÛåëî# Ýëççàâà äï äéðàéÞàÞ ðë ðàÛÝã ÝëèèñéäÝÛðäëé# ðäèà èÛéÛâàèàéð# ÛéÞ

ðë Üà ÛÝÝëñéðÛÜçà9

¸ ðë Üà ÛÝÝëñéðÛÜçà

¹ ÛÝÝëñéðÛÜäçäðö

º Üàäéâ ÛÝÝëñéðÛÜçà

» ðë ÛÝÝëñéð áëî ðãàèïàçòàï

q9 Ðäéïðëé ºãñîÝãäçç óÛï Ûé äéïìäîÛðäëéÛç ïìàÛæàî# Û ÜîäççäÛéð óÛî ïðîÛðàâäïð# ÛéÞ ãà çëòàÞ àôðîÛòÛâÛéð

áëëÞ9

¸ ãà çëòàÞ

¹ Û Ýëééëäïïàñî ëá

º ÛçóÛöï ÛììîàÝäÛðàÞ

» ãÛÞ Û ðÛïðà áëî

|9 Ðãàé ðãà æéëÝæ.æéëÝæ åëæà óÛï áäîïð äéòàéðàÞ äé ðãà Z¯qOï# ÝîäðäÝï ÝëèìçÛäéàÞ ðãÛð äð óÛï ÛÜïñîÞ#

áÛçïàçö äéðàççàÝðñÛç# ÛéÞ óëîîöäéâ ÛÜëñð äðï ëòàîóãàçèäéâ ìëìñçÛîäðö9

¸ óëîîöäéâ ÛÜëñð äðï ëòàîóãàçèäéâ ìëìñçÛîäðö9

¹ óàîà ñéãÛììö óäðã äðï ìëìñçÛîäðö9

º ëòàîçö ìëìñçÛî9

» óàîà óëîîöäéâ äð óÛï ðëë ìëìñçÛî9

‡9 Íãà èÛé çäáðàÞ ãäï Ýëèìëïð Üäé ÛéÞ ÞäïÝëòàîï Û ðãëñïÛéÞ óîäðãäéâ àÛîóäâï9

¸ çäáðàÞ ãäï Ýëèìëïð Üäé ÛéÞ ÞäïÝëòàîï

¹ çäáðàÞ ãäï Ýëèìëïð Üäé ÛéÞ óäçç ÞäïÝëòàî

º çäáðï ãäï Ýëèìëïð Üäé ÛéÞ ÞäïÝëòàîï

» çäáðï ãäï Ýëèìëïð Üäé ÛéÞ ÞäïÝëòàîàÞ

69
73
Parallelism Errors

1.
Forensic anthropologists recover remains, have examined physical characteristics to determine
geographical origins, and look for evidence of trauma.
A) NO CHANGE
B) examine
C) examined
D) are examining

2.
Regardless of a student’s major, college is intended to teach communication, time management,
and to be accountable.
A) NO CHANGE
B) accountability
C) being accountable
D) to account for themselves

3.
Winston Churchill was an inspirational speaker, a brilliant war strategist, and he loved food.
A) NO CHANGE
B) a connoisseur of
C) always appreciated
D) had a taste for

4.
When the knock-knock joke was first invented in the 1930s, critics complained that it was
absurd, falsely intellectual, and worrying about its overwhelming popularity.
A) NO CHANGE
B) were unhappy with its popularity.
C) overly popular.
D) were worrying it was too popular.

74
5.
The man lifted his compost bin and discovers a thousand writhing earwigs.
A) NO CHANGE
B) lifted his compost bin and will discover
C) lifts his compost bin and discovers
D) lifts his compost bin and discovered

6.
To land a job as a marketing agent, Joanne used a number of networking techniques, such as
joining professional trade groups, playing golf at courses, and hosted potlucks.
A) NO CHANGE
B) hosting
C) host
D) was hosting

7.
The professor had a reputation for being both dreary and didn’t like to wait.
A) NO CHANGE
B) not waiting.
C) being impatient.
D) impatient.

8.
Visitors to Arkansas’s Crater of Diamonds State Park are allowed to search for and keeping any
diamonds found on park grounds.
A) NO CHANGE
B) kept
C) to keeping
D) keep

75
9.
To protect the mountain gorillas of Virunga National Park requires a deep knowledge of the
forest, to appreciate these rare animals, and immense courage in the face of an ongoing conflict
in the Congo.
A) NO CHANGE
B) an appreciation of
C) appreciating
D) they must appreciate

10.
While still a caterpillar, the Large Blue butterfly gains entry to the nests of Myrmica ants by
falling to the ground, to pretend to be one of the ants’ own larvae, and hitching a ride back to the
colony.
A) NO CHANGE
B) pretending
C) pretends
D) can pretend

76
|9 ºëèìÛîäïëé ¼îîëîï

|.Z9 ºëñéðÛÜçà òï9 ÎéÝëñéðÛÜçà õÅÛéö D ÅñÝã# ½àó D Ääððçà

àô Á ãÛòà èñÝã Ûììçàï9 õÑ

|.e9 ÄëâäÝÛç ºëèìÛîäïëé õºãàÝæ äá öëñ Ûîà ÝëèìÛîäéâ àíñÛçï

àô Åö ÝÛî äï èëîà àôìàéïäòà ðãÛé ¹ëÜ9 õÑ

ÈîÛÝðäÝà Êñàïðäëéï

Z9 ÌäéÝà áàîèàéðàÞ ðëáñ îàíñäîàï Û ìîëçëéâàÞ ìàîäëÞ ëá ðäèà ðë áëîè# äð äï ëáðàé ïèàççäàî ðãÛé ëðãàî

áëîèï ëá áàîèàéðÛðäëé9

¸ ëðãàî áëîèï ëá áàîèàéðÛðäëé9

¹ ðëáñ áëîèàÞ Üö ëðãàî èàðãëÞï9

º ëðãàî èàðãëÞï ëá áëîèäéâ ðëáñ9

» ëðãàî ðëáñ.áëîèäéâ èàðãëÞï9

e9 Íãà Ä¿¹Í èëòàèàéð# óãäÝã ÝàçàÜîÛðàÞ ðãà çàâÛçä÷Ûðäëé ëá âÛö èÛîîäÛâà äé Âñéà eOZ‡# äï ëáðàé

ÝëèìÛîàÞ ðë ¸áîäÝÛé.¸èàîäÝÛéï ëá ðãà Z¯’Oï9

¸ ¸áîäÝÛé.¸èàîäÝÛéï

¹ ðãà ¸áîäÝÛé.¸èàîäÝÛé èëòàèàéð

º ¸áîäÝÛé.¸èàîäÝÛé ìàëìçà

» ¸áîäÝÛé.¸èàîäÝÛéäïè

q9 ¹àîàéäæÛêï îàéÞàîäéâ ëá ðãà ºãëìäé éëÝðñîéà óÛï èëîà äèìîàïïäòà ðãÛé ãàî ºãëìäé àðñÞà9

¸ ãàî ºãëìäé àðñÞà9

¹ ðãëïà ëá ðãà ºãëìäé àðñÞà9

º ðãÛð ëá ðãà ºãëìäé àðñÞà9

» ðãà ºãëìäé àðñÞà9

|9 Íãà óîäðäéâï ëá Àëéëîy Þà ¹Ûç÷ÛÝ äéÝçñÞà èëîà îàÛçäïðäÝ ÞàìäÝðäëéï ëá Z¯ðã Ýàéðñîö ÈÛîäïäÛé çäáà

ðãÛé Ûéö ½îàéÝã óîäðàî9

¸ Ûéö ½îàéÝã óîäðàî9

¹ ðãëïà ëá Ûéö ½îàéÝã óîäðàî9

º ðãÛð ëá Ûéö ½îàéÝã óîäðàî9

» Ûéö ëðãàî ½îàéÝã óîäðàî9

‡9 Íãà éñèÜàî ëá áàÛðãàîï ðãÛð Èäàîîà ìñð ëé ãäï ÍãÛéæïâäòäéâ ðñîæàö Ýëïðñèà ðãäï öàÛî óÛï çàïï ðãÛé

ðãëïà ëá çÛïð öàÛî9

¸ ðãëïà ëá çÛïð öàÛî9

¹ ðãÛð ëá çÛïð öàÛî9

º çÛïð öàÛîêï Ýëïðñèà9

» çÛïð öàÛîêï áàÛðãàîï9

71
77
Comparison Errors

1.
Since fermented tofu requires a prolonged period of time to form, it is often smellier than other
forms of fermentation.
A) NO CHANGE
B) tofu formed by other methods.
C) other methods of forming tofu.
D) other tofu-forming methods.

2.
Berenika’s rendering of the Chopin nocturne was more impressive than her Chopin etude.
A) NO CHANGE
B) those of the Chopin etude.
C) that of the Chopin etude.
D) the Chopin etude.

3.
The writings of Honoré de Balzac include more realistic depictions of 19th century Parisian life
than any French writer.
A) NO CHANGE
B) those of any French writer.
C) that of any French writer.
D) any other French writer.

4.
The number of feathers that Pierre put on his Thanksgiving turkey costume this year was less
than those of last year.
A) NO CHANGE
B) that of last year.
C) last year’s costume.
D) last year’s feathers.

78
5.
Yosef always told his sister more secrets than his brother, who was a real chatterbox.
A) NO CHANGE
B) he told his brother,
C) those of this brother,
D) his brother knew,

6.
Benjamin Van Horn’s first hybrid car, the Jolt, could not compete with Lindsay Carroll and her
first electric car, the Drift.
A) NO CHANGE
B) his competitor Lindsay Carroll.
C) Lindsay Carroll’s first electric car, the Drift.
D) Lindsay Carroll.

7. Pilar’s experiments in making gourmet hamburgers had mixed results. Her family agreed that
her bacon arugula burger was much better than cooking her sardine jam and peach burger.
A) NO CHANGE
B) concocting her sardine jam and peach burger.
C) her sardine jam and peach burger.
D) her invention of a sardine and jam burger.

8.
It can be challenging to explain the Japanese dance from “Butoh.” With a mixture of dramatic
visuals, absurdist movement, and silent theater, its storytelling techniques are dramatically
different than ballet or even modern dance.
A) NO CHANGE
B) those of ballet and modern dance.
C) ballet or modern dance.
D) how ballet and modern dance are choreographed.

79
9.
Though humans and birds both produce sound through the same physical mechanisms, they use
different organs to do so. A bird’s syrinx-produced vocalization is located much deeper in the
chest than its human counterpart, the larynx.
A) NO CHANGE
B) syrinx-produced vocalizations are
C) sounds, produced in the syrinx are
D) sound-producing syrinx is

10.
The park ranger recommended Julie, a seasoned hiker, a harder hiking trail than Wynona, a
complete novice.
A) NO CHANGE
B) Wynona’s hiking trail
C) that of Wynona
D) he did Wynona

80
‡9 ÅëÞäáäàî ¼îîëîï

‡.Z9 ÅëÞäáäàîï

A. ÅëÞäáäàîï° ÁéÝëèìçàðà ïàéðàéÝàï çÛÝæäéâ àäðãàî Û ïñÜåàÝð ëî Û òàîÜ9

1) ½îëéð ÅëÞäáäàî õÈãîÛïà#  ÌñÜåàÝð ÏàîÜ ÇÜåàÝð° ÈãîÛïà èëÞäáäàï Ì

àô Àäæäéâ ðãà ðîÛäç# ðãà ÜäîÞï ÝãäîìàÞ çëñÞçö9 õÑ

2) ¹ÛÝæ ÅëÞäáäàî õÌñÜåàÝð ÏàîÜ ÇÜåàÝð  ÈãîÛïà° ÈãîÛïà èëÞäáäàï Ç

àô Á áëñéÞ èö çëïð æàö óÛçæäéâ ðãîëñâã ðãà ìÛîæäéâ çëð9 õÑ

i. ¹ÛÝæ ÅëÞäáäàî ¼ôÝàìðäëé° ÌñÜåàÝð ÏàîÜ ÇÜåàÝð#  ÈãîÛïà° ÈãîÛïà èëÞäáäàï ÌÏÇ
àô Á ìÛïïàÞ ðãà àôÛè# èÛæäéâ èö ìÛîàéðï ãÛììö9 õÇ

ÈîÛÝðäÝà Êñàïðäëéï

Z9 Ãéëóé Üö èÛéö ìàëìçà Ûï ðãà óîäðàî ëá ÈöâèÛçäëé# ðãà ÝÛîààî ëá ¿àëîâà ¹àîéÛîÞ ÌãÛó ÜàâÛé äé

ðãàÛðàî ÝîäðäÝäïè9

¸ ðãà ÝÛîààî ëá ¿àëîâà ¹àîéÛîÞ ÌãÛó ÜàâÛé äé ðãàÛðàî ÝîäðäÝäïè9

¹ ðãà Üàâäééäéâ ëá ¿àëîâà ¹àîéÛîÞ ÌãÛó ÜàâÛé äé ðãàÛðàî ÝîäðäÝäïè9

º Üàäéâ Û ðãàÛðàî ÝîäðäÝ óÛï ðãà Üàâäééäéâ ëá ¿àëîâà ¹àîéÛîÞ ÌãÛóêï ìîëáàïïäëéÛç ÝÛîààî9

» ¿àëîâà ¹àîéÛîÞ ÌãÛó ÜàâÛé ãäï ìîëáàïïäëéÛç ÝÛîààî Ûï Û ðãàÛðàî ÝîäðäÝ9

e9 ¼ÞóÛîÞ ÌÛäÞä ÍäéâÛðäéâÛ# Û ðóàéðäàðã Ýàéðñîö Ûîðäïð áîëè ÍÛé÷ÛéäÛ# óãäÝã ÝëéðÛäéàÞ ïñîîàÛç

ìÛððàîéï ÛéÞ òäÜîÛéð Ýëçëî ÝãëäÝàï# óÛï æéëóé áëî ãäï çÛéÞïÝÛìà ìÛäéðäéâï9

¸ ÍäéâÛðäéâÛ# Û ðóàéðäàðã Ýàéðñîö Ûîðäïð áîëè ÍÛé÷ÛéäÛ# óãäÝã ÝëéðÛäéàÞ ïñîîàÛç ìÛððàîéï ÛéÞ òäÜîÛéð

Ýëçëî ÝãëäÝàï# óÛï æéëóé áëî ãäï çÛéÞïÝÛìà ìÛäéðäéâï9

¹ ÍäéâÛðäéâÛ óÛï æéëóé áëî ãäï çÛéÞïÝÛìà ìÛäéðäéâï# Û ðóàéðäàðã Ýàéðñîö ÍÛé÷ÛéäÛé Ûîðäïð# óãäÝã

ÝëéðÛäéàÞ ïñîîàÛç ìÛððàîéï ÛéÞ òäÜîÛéð Ýëçëî ÝãëäÝàï9

º ÍäéâÛðäéâÛ# ÝëéðÛäéäéâ ïñîîàÛç ìÛððàîéï ÛéÞ òäÜîÛéÝö# óÛï Û ðóàéðäàðã Ýàéðñîö Ûîðäïð áîëè ÍÛé÷ÛéäÛ

óãë óÛï æéëóé áëî ãäï çÛéÞïÝÛìà ìÛäéðäéâï9

» ÍäéâÛðäéâÛ# Û ðóàéðäàðã Ýàéðñîö ÍÛé÷ÛéäÛé Ûîðäïð# óÛï æéëóé áëî ãäï çÛéÞïÝÛìà ìÛäéðäéâï# óãäÝã

ÝëéðÛäéàÞ ïñîîàÛç ìÛððàîéï ÛéÞ òäÜîÛéð Ýëçëî ÝãëäÝàï9

q9 ¸çðãëñâã ìîëãäÜäðäòàçö àôìàéïäòà Ûð áäîïð# ïëèà çäÜîÛîäàï éëó ëóé ðãîàà.ÞäèàéïäëéÛç ìîäéðàîï#

äéÞñïðîäÛç îëÜëðï# óãäÝã ÝÛé ìîëÞñÝà Û ãñâà òÛîäàðö ëá ëÜåàÝðï9

¸ ïëèà çäÜîÛîäàï éëó ëóé ðãîàà.ÞäèàéïäëéÛç ìîäéðàîï# äéÞñïðîäÛç îëÜëðï# óãäÝã ÝÛé ìîëÞñÝà Û ãñâà

òÛîäàðö ëá ëÜåàÝðï9

¹ ðãîàà.ÞäèàéïäëéÛç ìîäéðàîï# äéÞñïðîäÛç îëÜëðï óãäÝã ÝÛé ìîëÞñÝà Û ãñâà òÛîäàðö ëá ëÜåàÝðï# Ûîà éëó

ÛòÛäçÛÜçà Ûð ïëèà çäÜîÛîäàï9

º ðãîàà.ÞäèàéïäëéÛç ìîäéðàîï Ûîà éëó ÛòÛäçÛÜçà Ûð ïëèà çäÜîÛîäàï# äéÞñïðîäÛç îëÜëðï óãäÝã ÝÛé ìîëÞñÝà

Û ãñâà òÛîäàðö ëá ëÜåàÝðï9

» ïëèà çäÜîÛîäàï# äéÞñïðîäÛç îëÜëðï óãäÝã ÝÛé ìîëÞñÝà Û ãñâà òÛîäàðö ëá ëÜåàÝðï# éëó ëóé ðãîàà.

ÞäèàéïäëéÛç ìîäéðàîï9

73
81
Modifier Errors

1.
Known by many people as the writer of Pygmalion, the career of George Bernard Shaw began in
theater criticism.
A) NO CHANGE
B) the beginning of George Bernard Shaw began in theater criticism.
C) being a theater critic was the beginning of George Bernard Shaw’s professional career.
D) George Bernard Shaw began his professional career as a theater critic.

2.
Edward Saidi Tingatinga, a twentieth century artist from Tanzania, which contained surreal
patterns and vibrant color choices, was known for his landscape paintings.
A) NO CHANGE
B) Tingatinga was known for his landscape paintings, a twentieth century Tanzanian artist,
which contained surreal patterns and vibrant color choices.
C) Tingatinga, containing surreal patterns and vibrancy, was a twentieth century artist from
Tanzania who was known for his landscape paintings.
D) Tingatinga, a twentieth century Tanzanian artist, was known for his landscape paintings,
which contained surreal patterns and vibrant color choices.

3.
Although prohibitively expensive at first, some libraries now own three-dimensional printers,
industrial robots, which can produce a huge variety of objects.
A) NO CHANGE
B) three-dimensional printers, industrial robots which can produce a huge variety of objects,
are now available at some libraries.
C) three-dimensional printers are now available at some libraries, industrial robots which
can produce a huge variety of objects.
D) some libraries, industrial robots which can produce a huge variety of objects, now own
three-dimensional printers.

82
4.
A rare form of currency, Ecuadorians have adopted the two-dollar bill as a good luck charm.
A) NO CHANGE
B) adoption of the two-dollar bill as a good luck charm has become a trend in Ecuador.
C) good luck if often associated with two-dollar bills in Ecuadorian culture.
D) the two-dollar bill has been adopted as a good luck charm by Ecuadorians.

5.
Cognitive neuroscientist Sophie Scott, an expert on laughter, suggests that it is more closely
related to animal calls and social interaction than speech or humor, a primitive noise.
A) NO CHANGE
B) Scott suggests that laughter is more closely related to animal calls and social interaction,
an expert on this primitive noise, than speech or humor.
C) Scott, an expert on laughter, suggests that this primitive noise is more closely related to
animal calls and social interaction than speech or humor.
D) Scott suggests that laughter is more closely related to animal calls and social interaction
than speech or humor, an expert on this primitive noise.

6.
Joining the list of edible objects that have traveled miles above the Earth, two Swedish men
attached a doughnut to a weather balloon.
A) NO CHANGE
B) a doughnut was attached to a weather balloon by two Swedish men.
C) a weather balloon was attached to a doughnut by two Swedish men.
D) the attachment between a doughnut and a weather balloon was conceived of by two
Swedish men.

7.
Serving her dessert on edible plates, Nicole was the star of the evening with her award-winning
chocolate soufflé cupcakes.
A) NO CHANGE
B) the award-winning chocolate soufflé cupcakes made Nicole the star of the evening.
C) the evening had a star: Nicole and her award-winning chocolate soufflé cupcakes.
D) the chocolate soufflé cupcakes served by Nicole were award-winning, making her the star
of the evening.

83
8.
Not allowed to attend school as a child due to religious gender restrictions, Begum Rokeya’s
science fiction later contained themes of gender equality.
A) NO CHANGE
B) gender equality became a theme in the science fiction written by Begum Rokeya.
C) Begum Rokeya later wrote science fiction with themes of gender equality.
D) the themes of Begum Rokeya’s later science fiction included gender equality.

9.
Scientists which most likely existed during the Jurassic Period from the Institute of Vertebrate
Paleontology and Paleoanthropology in China described a pigeon-sized dinosaur.
A) NO CHANGE
B) described a pigeon-sized dinosaur, which most likely existed during the Jurassic Period,
from the Institute of Vertebrate Paleontology and Paleoanthropology in China.
C) from the Institute of Vertebrate Paleontology and Paleoanthropology in China, which
most likely existed during the Jurassic Period, described a pigeon-sized dinosaur.
D) from the Institute of Vertebrate Paleontology and Paleoanthropology in China described a
pigeon-sized dinosaur, which most likely existed during the Jurassic Period.

10.
My family was excited with panda-like markings to visit The Staten Island Zoo and see the new
lamb.
A) was excited with panda-like markings to visit The Staten Island Zoo and see the new
lamb.
B) was excited to visit The Staten Island Zoo and see the new lamb with panda-like
markings.
C) was excited to visit The Staten Island Zoo with panda-like markings and see the new
lamb.
D) with panda-like markings was excited to visit The Staten Island Zoo and see the new
lamb.

84
’9 ÈñéÝðñÛðäëé

’.Z#e ÍÛÜçà õìâ9 ’’û’¯ Íöìàï ëá ÈñéÝðñÛðäëé

Íöìà ëá ÈñéÝðñÛðäëéï Íäìï

ÈàîäëÞï ß Ìàèä.Ýëçëé Æë ºëéåñéÝðäëé

ºëèèÛï ºëéåñéÝðäëé

ºëçëé ºçÛñïà ß ÈãîÛïà ÇÃ

»Ûïã »Ûïã.»Ûïã

àô Á ãÛÞ Þäééàî# ÛéÞ Á çàáð èö ãëèà9 õÑ

àô ÂÛéàï óÛéðï ðë Üà Ûé Ûðãçàðà# ïãà ìîÛÝðäÝàï ãÛîÞ9 õÑ

àô Á çäæà èÛéö ðöìàï ëá áîñäðï° Ûììçàï# ÜÛéÛéÛï# ÛéÞ ëîÛéâàï9 õÇ

àô Âëà — ÛéÞ ãäï áîäàéÞï — äï ÛçóÛöï óàçÝëèà9 õÇ

’.q ÍÛÜçà õìâ9 e ÏÛîäëñï ºëèèÛ ÎïÛâàï

ºëèèÛ ñïà Èñîìëïà

ÌÏÇ#  º9ºëéåñéÝðäëé  ÌÏÇ ºëééàÝð äéÞàìàéÞàéð ÝçÛñïàï

ÈãîÛïà#  ÌÏÇ ½îëéð èëÞäáäàî

ÌÏÇ#  ìãîÛïà ÈîëòäÞà ÛÞÞäðäëéÛç äéáëîèÛðäëé

Ì#  ìãîÛïà#  ÏÇ ÈãîÛïà èëÞäáäàï ÌñÜåàÝð


àô Á çäæà ãäè# Á Þëé ð çëòà ãäè9 õÑ

àô ½ààçäéâ ïäÝæ# èö ÝëèìÛéö âÛòà èà ïäÝæ çàÛòà9 õÑ

àô Á Ûè éëð Û âäîç# éëð öàð Û óëèÛé9 õÇ

àô ÅÛîö# æéëóé áëî ãàî ïãëëðäéâ ïæäççï# ïãëð ðóë ÜäîÞï äé ëéà ïãëð9 õÇ

’.|9 ¸ìëïðîëìãàï õÏàîÜDÈëïïàïïäòà

A. Äëëæ Ûð ÏàîÜ ß ÈçñîÛçäðö

àô Íãäï çÛóéèëóàîêï ÂÛéàêï Üàçëéâäéâïê9 õÑ

76
85
ÌäéâñçÛîDÈçñîÛç 쓰는 법
½ëî ðãà ìçñîÛç áëîè ëá èëïð éëñéï# ÛÞÞ ï9

● Üëððçà  Üëððçàï

● Ýñì  Ýñìï

½ëî éëñéï ðãÛð àéÞ äé Ýã# ô# ï# ëî ï ïëñéÞï# ÛÞÞ àï9

● Üëô  Üëôàï

● óÛðÝã  óÛðÝãàï

● Üñï  Üñïàï

½ëî éëñéï àéÞäéâ äé á ëî áà# ÝãÛéâà á ðë ò ÛéÞ ÛÞÞ àï9

● óëçá  óëçòàï

● çäáà  çäòàï

Ìëèà éëñéï ãÛòà Þäááàîàéð ìçñîÛç áëîèï9

● ÝãäçÞ  ÝãäçÞîàé

● èÛé  èàé

● èëñïà  èäÝà

● âëëïà  âààïà

Æëñéï àéÞäéâ äé òëóàçï çäæà ö ëî ë Þë éëð ãÛòà Þàáäéäðà îñçàï9

● ÜÛÜö  ÜÛÜäàï

● ðëö  ðëöï

● ìëðÛðë  ìëðÛðëàï

¸ áàó éëñéï ãÛòà ðãà ïÛèà ïäéâñçÛî ÛéÞ ìçñîÛç áëîèï9

● ïãààì  ïãààì

● Þààî  Þààî

● ïàîäàï  ïàîäàï

● ïìàÝäàï  ïìàÝäàï

77
86
Punctuation Errors

1.
Though it made many historical gains in space exploration, NASA’s Apollo Project is widely
known for one major accomplishment: landing the first humans on the moon.
A) NO CHANGE
B) accomplishment,
C) accomplishment;
D) accomplishment

2.
Morocco has had four capitals, collectively called the “Imperial Cities,” Fes, Marrakech,
Meknes,
and Rabat.
A) NO CHANGE
B) Cities”; Fes,
C) Cities”: Fes,
D) Cities” Fes

3.
Three animals are predicted to be the first genetically modified species approved for human
consumption, the “double-muscled” pig, fast-growing salmon, and hypoallergenic cows.
A) NO CHANGE
B) consumption;
C) consumption:
D) consumption, these are

87
4.
Women gained the right to vote in the United States in 1920; still, the U.S. lags behind other
countries such as: India, Costa Rica, and Ireland in electing a female head of state.
A) NO CHANGE
B) United States in 1920: still, the nation lags behind other countries such as India costa
Rica and Ireland
C) United states in 1920 still; the nation lags behind countries such as India, Costa Rica, and
Ireland
D) United States in 1920; still, the nation lags behind countries like India, Costa Rica, and
Ireland

5.
Sunsets on mars are not the familiar orange and red of those on Earth, the sky lights up with blue
and violet.
A) NO CHANGE
B) Earth; because
C) Earth;
D) Earth: because

6.
According to some sources, Carmen Sandiego had two motives; to fool young detectives and to
do “all the crimes.”
A) NO CHANGE
B) motives,
C) motives, both
D) motives:

7.
The wok pan is capable of many different uses; including stir-frying vegetables and braising
meats.
A) NO CHANGE
B) uses; these include
C) uses: including
D) uses, these include

88
8.
Sometimes when I’ve been working for hours, words start dancing before my eyes; this reminds
me that I need to take a break.
A) NO CHANGE
B) eyes; which reminds
C) eyes, reminding
D) eyes, which: reminds

9.
Because he was tired of eating take-out every night, Craig started baking bread once a week; as a
result, his relationship with his roommates—and later, his coworkers—flowered.
A) NO CHANGE
B) week; and
C) week:
D) week: and

10.
Vinaigrettes are easy to make, since all you need to do is mix three ingredients; vinegar, oil, and
spices.
A) NO CHANGE
B) ingredients:
C) ingredients, these are
D) ingredients: which are

89
9 ÌðîñÝðñîà

.Z9 ÁéÝëèìçàðà ÌàéðàéÝà

A. Äëëæ áëî ÌñÜåàÝð D ÏàîÜ

àô ¼ôìÛéÞäéâ ñéäòàîïà ïäéÝà ðãà ¹äâ ¹Ûéâ Z| Üäççäëé öàÛîï Ûâë9 õÑ

.e9 ºëéåñéÝðäëé ¼îîëî

A. ºëéåñéÝðäëéï° ÐëîÞï ðãÛð ÝëééàÝð ìÛîðï ëá ïàéðàéÝàï9

1) ¾ ëá ïñÜåàÝð. ¾ ëá ÝëéåñéÝðäëé ´ Z

àô ¸çðãëñâã ÂÛéà çäæàï Íëè# Üñð Íëè çäæàï ÅÛîö9 õÑ

ÍÛÜçà 9Z

Íöìàï ëá ºëéåñéÝðäëé ¼ôÛèìçàï

½¸Æ¹ÇÒÌ
ºëëîÞäéÛðäéâ ºëéåñéÝðäëé
õ½ëî# ¸éÞ# Æëî# ¹ñð# Çî# Òàð# Ìë

ÌñÜëîÞäéÛðäéâ ºëéåñéÝðäëé ¸çðãëñâã# ¸áðàî# Ðãàðãàî# Ðãàé# Ðãë# àðÝ9

.q9 ¸ÞòàîÜ ºçÛñïà

ÍÛÜçà 9e Íöìàï ëá ¸ÞòàîÜ ºçÛñïàï

õÌðñÞàéðï ïãëñçÞ Üà ÛÜçà ðë ðàçç ðãà ÞäááàîàéÝà Ûèëéâ ìîàìëïäðäëéï# ÝëéåñéÝðäëéï# ÛéÞ ÝëéåñéÝðäòà

ÛÞòàîÜï

Íäèà.îàçÛðàÞ óãàé# Üàáëîà# ïäéÝà# ñéðäç# Ûáðàî# Üö ðãà ðäèà

ËàÛïëé ÜàÝÛñïà# ïäéÝà

ºëéðîÛïð Ûçðãëñâã# ðãëñâã# àòàé ðãëñâã

ºëéÞäðäëéÛç äá# ñéçàïï

Çðãàîï äé ðãÛð# äé ÛïèñÝã Ûï

A. ¸ÞòàîÜ ºçÛñïà äé áîëéð ëá ðãà èÛäé ÝçÛñïà

1) Îïà ÝëèèÛ Üàáëîà ðãà èÛäé ÝçÛñïà

B. ¸ÞòàîÜ ºçÛñïà Ûáðàî ðãà èÛäé ÝçÛñïà

1) ºëèèÛ äï éëð éàÝàïïÛîö

àô ¸çðãëñâã Á Ûè ëòàîóàäâãð Á îñé ìîàððö áÛïð9 õÑ

80
90
.|9 ºëéåñéÝðäëé òï9 ºëéåñéÝðäòà ¸ÞòàîÜ

ÍÛÜçà 9q Íöìàï ëá ºëéåñéÝðäòà ¸ÞòàîÜï

õÌðñÞàéðï ïãëñçÞ Üà ÛÜçà ðë ðàçç ðãà ÞäááàîàéÝà Ûèëéâ ìîàìëïäðäëéï# ÝëéåñéÝðäëéï# ÛéÞ ÝëéåñéÝðäòà

ÛÞòàîÜï

Ëàïñçð ðãàîàáëîà# ðãñï# Ýëéïàíñàéðçö# ÛÝÝëîÞäéâçö# ðãàîàÜö# ãàéÝà

ºëèìÛîäïëé çäæàóäïà# ïäèäçÛîçö# ÝëèìÛîÛðäòàçö

ãëóàòàî# äéïðàÛÞ# îÛðãàî# ïðäçç# öàð# ÝëéðîÛîäçö# éàòàîðãàçàïï#


ºëéðîÛïð
éëéàðãàçàïï

ÌàíñàéÝà áäéÛççö# éàôð# ðãàé# çÛïðçö# áäîïð# ïàÝëéÞ# ðãäîÞ

¸ÞÞäðäëé ÛâÛäé# ÜàïäÞàï# áñîðãàîèëîà# äéÞààÞ# èëîàëòàî# Ûçïë

A. ºëéåñéÝðäòà ÛÞòàîÜï ÝÛééëð îàìçÛÝà ÝëéåñéÝðäëéï äé Û ïàéðàéÝà9

àô Á çäæàÞ ðãà ÝÛî# ãëóàòàî# ïëèàÜëÞö ÛçîàÛÞö Üëñâãð äð9 õÑ

ÈîÛÝðäÝà Êñàïðäëéï

Z9 ÌäéÝà ðãà ÞäïÝëòàîö ëá ºãäéÛêï ÍàîîÛÝëððÛ ¸îèö äé Z¯|# ÛîÝãÛàëçëâäïðï ãÛòà ïðñÞäàÞ ðãà #OOO

ïðÛðñàï# ãëóàòàî# ðãàö ãÛòà éëð öàð ÞàðàîèäéàÞ äá àÛÝã ïðÛðñà óÛï ÜÛïàÞ ëé Ûé ÛÝðñÛç ìàîïëé9

¸ ãÛòà ïðñÞäàÞ ðãà #OOO ïðÛðñàï# ãëóàòàî# ðãàö ãÛòà éëð öàð ÞàðàîèäéàÞ

¹ ãÛòà ïðñÞäàÞ ðãà #OOO ïðÛðñàï# Üñð ãÛòà éëð öàð ÞàðàîèäéàÞ

º ãÛòà ïðñÞäàÞ ðãà #OOO ïðÛðñàï Üñð ðãàö ãÛòà éëð öàð ÞàðàîèäéàÞ

» ãÛòà ïðñÞäàÞ ðãà #OOO ïðÛðñàï Üñð ãÛòà éëð öàð ÞàðàîèäéàÞ

e9 ¸çðãëñâã ðãàö Ûîà Üàððàî.æéëóé áëî ðãàäî èäçäðÛîö ÝÛîààîï Þñîäéâ ðãà ¸èàîäÝÛé ºäòäç ÐÛî# Üñð

ÌðëéàóÛçç ÂÛÝæïëé ÛéÞ ËëÜàîð ¼9 Äàà áäîïð ïàîòàÞ ðëâàðãàî äé ðãà ÅàôäÝÛé.¸èàîäÝÛé ÐÛî9

¸ ¸èàîäÝÛé ºäòäç ÐÛî# Üñð ÌðëéàóÛçç ÂÛÝæïëé ÛéÞ ËëÜàîð ¼9 Äàà

¹ ¸èàîäÝÛé ºäòäç ÐÛî# ÌðëéàóÛçç ÂÛÝæïëé ÛéÞ ËëÜàîð ¼9 Äàà

º ¸èàîäÝÛé ºäòäç ÐÛî# ãëóàòàî# ÌðëéàóÛçç ÂÛÝæïëé ÛéÞ ËëÜàîð ¼9 Äàà

» ¸èàîäÝÛé ºäòäç ÐÛî# ðãà ðóë# ÌðëéàóÛçç ÂÛÝæïëé ÛéÞ ËëÜàîð ¼9 Äàà# ãÛòäéâ

q9 ÅÛîäéà Üäëçëâäïðï ïðÛðà ðãÛð àéÞÛéâàîàÞ ÝëîÛç îààáï ÝÛé îàÜëñéÞ# ïë ðãÛð ìàëìçà ÝÛé èÛæà Ûé àááëîð

ðë ÞàÝîàÛïà ëòàîáäïãäéâ# ìëççñðäëé# ÛéÞ âçëÜÛç ðàèìàîÛðñîàï9

¸ ïë ðãÛð

¹ ìîëòäÞàÞ ðãÛð

º àòàé ðãëñâã

» ñéçàïï

|9 Ðãäçà ðãà ÈÛçè ÁïçÛéÞï äé »ñÜÛä# Î9¸9¼ óàîà äéäðäÛççö àéòäïäëéàÞ Ûï ðîäÜñðàï ðë çñôñîö# ðãà ìîëåàÝð

ãÛï Üààé áîÛñâãð óäðã áäéÛéÝäÛç ÛéÞ àéòäîëéèàéðÛç ìîëÜçàèï ïäéÝà äðï âîëñéÞÜîàÛæäéâ9

¸ Ðãäçà ðãà ÈÛçè ÁïçÛéÞï äé »ñÜÛä# Î9¸9¼9 óàîà äéäðäÛççö àéòäïäëéàÞ Ûï ðîäÜñðàï ðë çñôñîö#

¹ Íãà ÈÛçè ÁïçÛéÞï äé »ñÜÛä# Î9¸9¼9 óàîà äéäðäÛççö àéòäïäëéàÞ Ûï ðîäÜñðàï ðë çñôñîö#

º Íãà ÈÛçè ÁïçÛéÞï äé »ñÜÛä# Î9¸9¼9 óàîà äéäðäÛççö àéòäïäëéàÞ Ûï ðîäÜñðàï ðë çñôñîö# ãëóàòàî#

» Ðãäçà ðãà ÈÛçè ÁïçÛéÞï äé »ñÜÛä# Î9¸9¼9# äéäðäÛççö àéòäïäëéàÞ Ûï ðîäÜñðàï ðë çñôñîö#

81
91
Structure Errors

1.
Since the discovery of China’s Terracotta Army in 1974, archaeologists have studied the 7,000
statues, however, they have not yet determined if each statue was based on an actual person.
A) NO CHANGE
B) have studied the 7,000 statues, but have not yet determined
C) have studied the 7,000 statues but they have not yet determined
D) have studied the 7,000 statues but have not yet determined

2.
Although they are better-known for their military careers during the American Civil War, but
Stonewall Jackson and Robert E. Lee first served together in the Mexican-American War.
A) NO CHANGE
B) American Civil War, Stonewall Jackson and Robert E. Lee
C) American Civil War, however, Stonewall Jackson and Robert E. Lee
D) American Civil War, the two, Stonewall Jackson and Robert E. Lee, having

3.
Marine biologists state that endangered coral reefs can rebound, so that people can make an
effort to decrease overfishing, pollution, and global temperatures.
A) NO CHANGE
B) provided that
C) even though
D) unless

4.
While the Palm Islands in Dubai, U.A.E were initially envisioned as tributes to luxury, the
project
has been fraught with financial and environmental problems since its groundbreaking.
A) NO CHANGE
B) The Palm Islands in Dubai, U.A.E. were initially envisioned as tributes to luxury,
C) The Palm Islands in Dubai, U.A.E. were initially envisioned as tributes to luxury,
however,
D) While the Palm Islands in Dubai, U.A.E., initially envisioned as tributes to luxury,

92
5.
The TV show Mozart in the Jungle provides a funny, farcical look at classical musicians in a
New York symphony; yet, for musicians, the show has become a hit because they find it
relatable.
A) NO CHANGE
B) New York symphony; still,
C) New York symphony, still,
D) New York symphony but

6.
Ocean transportation, inasmuch as air or roadway portage, carries the most commercial cargo
worldwide.
A) NO CHANGE
B) because of
C) rather than
D) whether

7.
Rupert swore that he’d never seen anything so beautiful, so radiant, so positively divine as the
glorious triple layer ice cream cake, however, he may have been exaggerating.
A) NO CHANGE
B) ice cream cake, but he may have been exaggerating.
C) ice cream cake, rather, a big exaggeration.
D) ice cream cake, but an exaggeration.

8.
Contrary to popular belief, the majority of sugar produced in the United States does not come
from sugar canes but is obtained from sugar beets.
A) NO CHANGE
B) the majority of sugar produced in the United states, coming from sugar beets rather than
sugar canes.
C) nevertheless, the majority of sugar produced in the United States does not come from
sugar canes but is obtained from sugar beets.
D) although the majority of sugar produced in the United States does not come from sugar
canes but actually from sugar beets.

93
9.
There are several differences between European and Asian pears, for example, the former
sometimes change color after being harvested.
A) NO CHANGE
B) Asian pears; for example, the former
C) Asian pears, the former
D) Asian pears; like the former

10.
The Chauvet Cave in France features the world’s oldest cave paintings. In order to preserve the
paintings, only a few people may view them every year.
A) NO CHANGE
B) In order to preserve the paintings, so only a few people may view them every year.
C) Preserving the paintings, only a few people may view them every year.
D) Only a few people may view the paintings every year, since to preserve the paintings.

94
SAT
Writing Test 1

83
95
96
Q u
e
s
t
i
o
n
s
1
-
1
1
a
r
e
b
a
s
e
d
o
n
t
h
e
f
o
l
l
o
w i
n
g
p
a
s
s
a
g
e
.
1

Tomorrow's Classroom A) NO CHANGE


B) with, a room
Here's a scene you probably are extremely familiar C) with a room,
D) with a room
with: a room full of high school students sitting

patiently in a classroom as the teacher explains a

concept from one of seven or eight subjects. This

scene has been experienced by humans in the

developed world on a standard basis for almost 2


200 years, but may be about to change. As the nature A) NO CHANGE
B) commonplace
of work, government, and society itself changes, so
C) prevalent
too will education. Long considered an admirable but 3 D) regular

not very prestigious career, teaching might prove to be

a more dynamic job in the coming years.


3

2
2

85
97
Historically speaking, the concept of public education 3
is a relatively new one, and it can be traced back to A) NO CHANGE
B) would have been linked
both the philosophical trends of the Enlightenment in
C) was linked
the 18th century and the increased need for skilled D) had linked

labor that would be linked to the Industrial

Revolution. Factories required workers who could do more 4


than simple physical labor. Also, the push for public A) NO CHANGE
B) while
education and democracy went hand in hand, and in
C) but
order for democracy to function properly, people needed 3 D) because

to understand what they were participating in and trying

to change. In the 19th and 20th centuries, these factors

led to the rise of public education. This was funded 5

by governments that sought to produce an educated Which choice most effectively combines the
underlined sentences?
workforce. The system worked well and led to overall A) In the 19th and 20th centuries, these factors
led to the rise of public education, which was
increases in literacy and education levels.
funded by governments that sought to produce
an educated workforce.
B) In the 19th and 20th centuries, these factors led
to the rise of public education, and this rise
was funded by governments that sought to
produce an educated workforce.
C) Funded by governments that sought to produce
an educated workforce, these factors led to the
rise of public education in the 19th and 20th
centuries.
D) Funded by governments, these factors led to
the rise of public education in the 19th and
20th centuries, and they sought to produce an
educated workforce.

2
3

86
98
However, as both society and the economy have 6
developed, our systems of public education have struggled A) NO CHANGE
B) match up.
to even out. The schools of the 20th century were
C) keep pace.
geared to produce competent manufacturing employee’s D) go toe-to-toe.

and laborers’, but trends in these industries toward

artificial intelligence and automation have altered the


7
students’ expectation of what education is supposed to
A) NO CHANGE
provide. Rather than the traditional reading, writing, and B) employees and laborers,
C) employee’s and laborer’s,
arithmetic that are provided by public education, the D) employees’ and laborers,
3
modern world now requires reasoning, problem solving,

and creativity. Parents, students, and employers alike are

frustrated at education’s delayed response to society’s


8
changing needs.
Which choice most effectively sets up the sentence
that follows?
A) NO CHANGE
B) were representative of the explosive growth of
Information Technology (IT).
C) have sharply shifted the needs of business
from physical to intellectual labor.
D) have articulated a dire need for reform in
the training of educational professionals.

9
Which choice provides the best transition to the
next paragraph?
A) NO CHANGE
B) looking outside of traditional education for
solutions.
C) calling for an overhaul to the archaic,
inefficient system.
D) increasingly relying on the private sector for
new innovations.

2
4

87
99
As a result, opportunities for creative educators are 10
on the rise. The technology that has diminished the need Which choice most effectively supports the claim
made in the previous sentence?
for manufacturing work has also served to broaden the
A) NO CHANGE
possibilities for education. In the age of the Internet, B) To be an effective educator in the 21st
century, it is important to cover a wide range
teachers are moving beyond the confines of the traditional of disciplines
classrooms. Virtual classrooms allow teachers to C) A recent study found that more and more
students are taking classes online
collaborate with wider groups of students—all without D) The main reason, of course, is the privatization
of education
pencils, papers, or blackboards. Teachers, parents, and

students can decide on the best curriculum that coincides

with the needs of society. At the same time, there is no

need to completely dismantle the existing system:

instead, the new system can coexist with the old. The

possibilities are endless.

11
A) NO CHANGE
B) system;
C) system—
D) system,

2
5

88
100
Q u
e
s
t
i
o
n
s
1
2
-
2
2
a
r
e
b
a
s
e
d
o
n
t
h
e
f
o
l
l
o 12
w i
n
g
p
a
s
s
a
g
e
.
A) NO CHANGE
Quantum Biology B) distinct, and numerous tasks, they
C) distinct and numerous tasks, they
According to basic biology, the human body is D) distinct, numerous tasks they

composed of cells. Although these cells specialize in

distinct and numerous tasks they function together

as a living organism—whether it be a worm, a bird,

or a human being. An organism’s cellular processes work


13
A) NO CHANGE
according to chemical laws that are complex but still
B) that can be predicted.
follow predictable patterns that can be anticipated. C) that can be expected.
D) DELETE the underlined portion and end the
The fact that atoms have positive and negative charges 3 sentence with a period.
depending on the presence or absence of electrons,

combine with the properties of particular elements,

allows the functions of cells. Simply put, chemistry plus 14


biology equals life. However, while biology has revealed A) NO CHANGE
B) combined
much about the functions of life, there are still many
C) combines
mysteries, and some researchers are delving into D) was combining

unusual places in the search for answers—namely,

quantum physics.

15
A) NO CHANGE
B) digging inside
C) taking note of
D) going in

2
6

89
101
Quantum physics is far too complex to explain briefly 16
and accurately, but one way to think about it is in the A) NO CHANGE
B) scale, which is far, far smaller outcomes,
difference in scale between classical and quantum physics.
C) scale—which is far, far smaller—outcomes
"In classical physics, particles like electrons behave in D) scale: which is far, far smaller, outcomes

predictable and binary, on-off ways," said Dr. Julia Jackson,

a researcher at the Department of Biophysics at MIT. "At

the quantum scale, which is far, far smaller outcomes

are less predictable but have more possible outcomes than


17
on or off." This uncertainty enables greater efficiency in
A) NO CHANGE
energy transfer and the kinds and amounts of information B) observed but can only be proved
C) observed. Which can only be proved
that can be handled by biological processes. The only
D) observed, but can only be proved
problem is that these processes are happening at such a

small, delicate level that they cannot be directly

observed, they can only be proved by logical

inference.

16

17

18

2
7

90
102
One of the biggest mysteries of biology that 18
might be explained by quantum processes is one of Which choice provides the best transition from the
previous paragraph to this one?
life's oldest and most common phenomena: photosynthesis.
A) NO CHANGE
Plants use photosynthesis to convert sunlight into energy B) Literature has identified ancient algae as the
forefather of a prevalent biological phenomenon:
that they can use for biological processes, but the mystery photosynthesis.
is exactly how they do it so well. "Photosynthesis is 99% C) For all organisms—whether they are plants or
animals—securing energy is of primary
efficient," said Dr. Jackson. "According to conventional importance.
electrical diffusion principles, such a high conversion rate D) Quantum physics and biology may be related in
many more ways than people realize.
should not be possible." If quantum biology is involved,

for example, it is a different story. The

chloroplast cells may incorporate excited quantum states.


19
These states release their energy only when needed,
A) NO CHANGE
enabling extraordinary efficiency. B) subsequently,
C) however,
3 D) conversely,

20
Which choice most effectively combines the
underlined sentences?
A) The chloroplast cells may incorporate excited
quantum states; these states release energy only
when needed, which enables extraordinary
efficiency.
B) The chloroplast cells may incorporate excited
quantum states: the result being that they
release energy only when needed, enabling
extraordinary efficiency.
C) The chloroplast cells may incorporate excited
quantum states, and the energy released only
when needed enables extraordinary efficiency.
D) The chloroplast cells may incorporate excited
quantum cells that release energy only when
needed, enabling extraordinary efficiency.

2
8

91
103
Not all chloroplasts exhibit the principles of 21
quantum uncertainty, however. Both vision and olfaction Which choice most effectively reinforces one of the
passage’s central claims about the mysterious nature
(the sense of smell) operate at extremely efficient rates that
of biological processes?
have long puzzled classical biologists. Recent research of A) NO CHANGE
B) Of course, the field of quantum biology is yet
quantum biologists, meanwhile, shows that these processes in its infant stage, with many questions
can be explained by quantum states existing within the cell remaining unanswered.
C) Exploring the processes that might involve
structures of the eyes and nose. quantum effects shows not only how
widespread biological quantum phenomena
Dr. Jackson is excited for the future of the field.
might be but also how many important aspects
"The more we explore this area, the more possibilities of life remain little understood.
D) Despite their continued collaborations, physicists
become apparent," she said. "If we can truly grasp how and biologists remain mostly separate.
these processes work, then who knows what practical and

medical applications might become possible." 22


At this point, the writer wants to add a second
example that demonstrates the relevance of quantum
processes to the field of biology. Which choice best
accomplishes this goal?
A) Biologists are now working in tandem with
physicists to aid their understanding of difficult
molecular processes.
B) Even further, certain migratory animals such as
birds and foxes show the ability to navigate
using the earth's magnetic field, a process that
may also be related to quantum states.
C) If eyes and noses are influenced by quantum
processes, then it is only logical to assume
other organs may be, as well.
D) Although quantum physics is at its inception, it
can learn a lot from the rigors of molecular
biology.

2
9

92
104
Q u
e
s
t
i
o
n
s
2
3-
3
3ar
e
b
a
s
e
d
o
n
t
h
e
f
o
l
l
o
w i
n
g
p
a
s
s
a
g
e
.
23
A) NO CHANGE
The Golden Life of Mansa Musa B) his or her
C) one’s
What is your deepest desire? The possible answers to
D) your
this question are limitless, but the most common answers

are success, love, and fame. Another common answer is

wealth. But what does it mean to be wealthy? How does 24

one measure the relative value of their possessions? The A) NO CHANGE


B) Musa was
life of Malinese king Mansa Musa, he was widely C) his being
3 D) DELETE the underlined portion.
considered the wealthiest single human being to ever live,

might serve as an example of what can happen when one

gets everything out of life. From the beginning of his reign


25
in 1312 until his death in 1337: Mansa Musa
A) NO CHANGE
represented the pinnacle of human prosperity. B) 1337 Mansa Musa, represented
Mansa Musa ruled over what was arguably the most C) 1337, Mansa Musa, represented
D) 1337, Mansa Musa represented
prosperous area of the world at the time. From the

twin metropolises of Timbuktu and Gao, Mansa Musa

managed a trade network across the Sahara that connected


26
to Tunisia, Egypt, and beyond.
At this point, the writer is considering adding the
following sentence.

While much of the world struggled with


disasters like the bubonic plague and the Mongol
invasions, the west African kingdom of Mali
thrived as a center of trade.

Should the writer make this addition here?


A) Yes, because it provides a context in which
Mali’s prosperity was notable.
B) Yes, because it establishes an important shift in
the paragraph’s discussion of Mali’s wealth.
C) No, because it interrupts the paragraph’s
discussion with irrelevant information.
D) No, because it repeats information that is
already presented earlier in the passage.

30

93
105
In Mansa Musa’s time, gold was the standard currency, 27
and there was no banking system. Indeed, wealth was A) NO CHANGE
B) Therefore,
measured by how much gold one possessed, and Mansa
C) For example,
Musa possessed the largest and most productive gold mines D) However,

of the Late Middle Ages (years 1300-1500). In fact, he had

so much gold relative to other civilizations that he is widely

considered to be the wealthiest human being of all time, 28


even after adjusting for inflation. Some estimates put his A) NO CHANGE
wealth in 2017 dollars at $500 billion, $200 billion B) more then
C) more than
more than that of the next wealthiest person in history, 3 D) more than those of

US industrial monopolist Nelson D. Rockefeller.

Mansa Musa’s wealth was on a scale that could

only be explained by his religious affiliation. A devout 29

Muslim, Musa decided to go on a pilgrimage across Which choice best supports the main point of the
passage?
thousands of kilometers of Saharan desert to Mecca in A) NO CHANGE
B) that only the richest of European monarchs
1324. With him he took an enormous caravan of court
could match.
followers on camelback. Contemporary witnesses said the C) commensurate with Musa’s influence as a
benevolent ruler.
procession stretched as far as the eye could see,
D) that was entirely unimaginable by commoners
extending for a long distance. Given the remote and rulers of other societies alike.

location of Mali relative to the rest of Europe and Asia,

many did not even know Mansa Musa existed until


30
he showed up in Egypt on his way to Mecca.
A) NO CHANGE
B) which extended for a long distance.
C) and it extended very far.
D) DELETE the underlined portion and end the
sentence with a period.

29

31

94
106
He spent so much gold on provisions and in gifts to 31
the poor that the value of gold descended in the A) NO CHANGE
B) swooped down
region for a generation. Word of his wealth and power
C) plummeted
spread, and European mapmakers began to include him D) appreciated

with such stature on their maps that he appeared as large

as a force of nature, like a sea or mountain range.

[1] If the only thing he ever accomplished was to gain

an immense fortune, Mansa Musa would still be notable. 32


[2] He provided services for the poor and built extensive A) NO CHANGE
B) environment
improvements to the cities within his kingdom, many of
C) surroundings
which still stand today, like the University of Sankore or D) colossi

the Djinguereber Mosque. [3] The legacy left by Mansa 3


Musa was more than buildings and wealth. [4] However, he

was also by all accounts a wise and conscientious ruler.

[5] He stood as an example of the proper use of the power

that comes with great success. 33


To make this paragraph most logical, sentence 4
should be
A) placed where it is.
B) placed after sentence 1.
C) placed after sentence 2
D) DELETED from the paragraph.

33

32

95
107
Questions 23-33 are based on the following passage 23
and supplementary material.
Which choice is the best introduction to the
paragraph?
The Rise of Podcasting A) NO CHANGE
B) In the 20th century, radio was one of the
The ubiquity of radio has remained unchallenged dominant media formats.
C) Despite the rise of new media, radio is still a
throughout the 20th century. Before television and film, beloved mode of public communication.
families used to gather around there radios to hear D) The changing technology of the 21st century is
quickly replacing an age-old pastime.
serialized audio stories, musical performances, and news

reports. Because it is almost free of cost and available in

all automobiles, radio is still widely popular; but the 35


artistic audio forms that used to be found only with the A) NO CHANGE
turn of the dial are now migrating to an upgraded B) their
C) they’re
medium— the podcast. Since 2009, the podcast has D) one’s
exploded in popularity and diversity and is now

bursting with possibilities for future development. 36

This success was by no means guaranteed, however. A) NO CHANGE


B) popular, but,
When podcasts were first introduced, they were limited by C) popular, but
D) popular: and
the technology that gave them their name, the iPod — an

mp3 audio device manufactured by the Apple corporation.

With the introduction of updated podcasting software, 37


however, users were able to record, upload, and A) NO CHANGE
B) saturated with
automatically distribute audio files to anyone who wanted C) jam-packed with
to subscribe. D) filled with

33

96
108
The resulting content was not unlike the radio, 38
except that users were able to control when and where A) NO CHANGE
B) that for the
they listened. If there lingered a minor but crucial
C) that of the
inconvenience of having to transfer the audio files, D) that from the

podcasts quickly became a preferred form of media

among the tech-savvy. 39

With the decline of the dedicated music devices like A) NO CHANGE


B) Until
iPods, many in the industry predicted an end of an era— C) Although
3 D) Unless
that the podcast would lose its popularity, representing a

mere footnote in the history of media. Able to

download or stream programs from anywhere at anytime,


40
the popularity of podcasts exploded and transcended At this point, the writer is considering adding the
the limitations of dedicated music players. News programs, following sentence.

comedy shows, musical performances, and particularly In fact, the opposite occurred.

audio stories began to develop rapidly. Podcast-only Should the writer make this addition here?
A) Yes, because it provides an effective transition
programming became sensational hits. The popularity of to the discussion of podcasts’ popularity despite
Serial—a long form, true-crime story—cemented the the prediction.
B) Yes, because it restates the writer's central
podcast as a mainstay of digital media. claim of the passage.
C) No, because it does not elaborate the reason
why the popularity of podcasts increased.
D) No, because it contradicts the writer's main
claim in the paragraph.

41
A) NO CHANGE
B) the smartphones actually played a pivotal role
in securing the podcast’s place in modern
media landscape.
C) the podcasts, instead, shot up in popularity in
an unprecedented manner.
D) the era of podcasts seemed to be upon
dedicated listeners.

34

97
109
As of 2017, more than 67 million Americans listen to 42
podcasts on a monthly basis (a figure that surpasses the A) NO CHANGE
B) month), although
number of moviegoers in a month), the average
C) month), and
American spends 4.5 hours a week listening to them. D) month), instead,

Advertisers spent $34 million on podcasts in 2017

alone, they attested to their new popularity.


43
Furthermore, these numbers are expected to increase
A) NO CHANGE
over the next 10 years. "It's the start of a new art form B) alone, their attesting
C) alone; attesting
in many ways," said Scott Aukerman of the well-known 3 D) alone, attesting
podcast Comedy Bang! Bang! "Even those of us, who

have been in it from the beginning are excited about


44
the recent growth."
At this point, the writer is considering adding the
following sentence.

In fact, according to the data presented in the


graph, podcasts overtook AM/FM radio as the
dominant medium in 2016.

Should the writer make this addition here?


A) Yes, because it provides additional support for
the main point of the paragraph.
B) Yes, because it addresses a possible
counterargument to the writer's main claim
C) No, because it is not an accurate interpretation
of the data.
D) No, because it introduces irrelevant information
that interrupts the flow of the passage.

F
igm Ra
u
r
e
a
d
a
p
t
e
d
f
r
o d
i
o
We
e
k
ly,
2
0
1
8 44

STOP
If you finish before time is called, you may check your work on this section only.

Do not turn to any other section.

35

98
110
SAT
Writing Test 2

99
111
112
Q u
e
s
t
i
o
n
s
1
-
1
1
a
r
e
b
a
s
e
d
o
n
t
h
e
f
o
l
l
o
w i
n
g
p
a
s
s
a
g
e
.
1
In context, which choice best combines the
Your Home and Office, Redesigned underlined sentences?
A) The nature of labor continues its transition
The relationship between humans and the spaces from work of the hand to work of the mind.
So too are our workspaces transition from
they inhabit changes over time. The nature of industrial-style facilities into places that are
more supportive of human creativity.
labor continues its transition from work of the hand
B) The nature of labor continues its transition
to work of the mind. Our workspaces transition from work of the hand to work of the mind
and our workspaces transition from
from industrial-style facilities into places that are industrial-style facilities into places that are
more supportive of human creativity.
more supportive of human creativity. Not only at
C) When the nature of labor continues its
work but also at home, we search for ways to transition from work of the hand to work of
the mind, our workspaces transitioning from
increase the harmony and beauty of our daily lives. industrial-style facilities into places that are
more supportive of human creativity.
D) As the nature of labor continues its transition
from work of the hand to work of the mind,
so too are our workspaces transitioning from
industrial-style facilities into places that are
more supportive of human creativity.

3
2
2

101
113
It is not a new profession, but as people begin to 2
demand better design from the world around them, A) NO CHANGE
B) smarter, and more talented
they will also demand better, smarter, more C) smarter, and are more talented
D) smarter, being more talented
talented designers.

The concept of interior design first arose with the 3


modern concept of the middle class in the 1880s. A) NO CHANGE
B) from
Visible status became more socially important as 3 C) through
increasing numbers of people had money to spend D) DELETE the underlined portion.

on luxuries. Homes and offices were used as means 4

of showing off their wealth. The first interior A) NO CHANGE


B) In fact,
design was done by large furniture companies that C) Finally,
D) As a result,
would sell entire sets of pre-arranged furniture for

homes and offices. Later, individuals began to offer 5


private, individualized services that allowed people to A) NO CHANGE
B) whose
customize their spaces to suit their personalities, and C) who's
the work quickly developed into its own artistic field. D) that

However, some of the first work ever done in 6


the field of design was done by these early interior At this point, the writer is considering adding the
following sentence.
designers. Owen Jones, his book The Grammar
He looked towards the Islamic world for much of
of Ornament is one of the earliest and most influential his inspiration, using his carefully observed
books about graphic design, was employed by multiple studies of Islamic decoration at the Alhambra to
develop bold new theories on flat patterning,
interior design firms. geometry, and abstractions in ornament.

Should the writer make this addition here?


A) Yes, because it adds useful data that helps to
better understand the motivation for his studies.
B) Yes, because it supplies information that
supports the ideas referred in the next
paragraph.
C) No, because the information added interrupts
the discussion of the main ideas in the passage.
D) No, because it does not take into account
whether his ideas were inspired by other works
than Islamic world.

2
3

102
114
In the modern day, the importance of antiques 7
Which choice provides the most appropriate
and rare goods extend beyond the aesthetic and
introduction to the paragraph?
expressive into the practical and productive. According to A) NO CHANGE
B) the importance of good interior design
the Bureau of Labor Statistics, during the last decade, C) the importance of luxuries and modern
poor interior design and ergonomics annually resulted furnitures
D) living in a prestigious mansion
in over $100 billion of direct and indirect costs to

businesses each year. Employees are often not aware


8
of or not provided with the space or materials they A) NO CHANGE
need to avoid repetitive stress injuries, inefficiencies, B) extended
3 C) extends
and distractions, resulting in a marked decrease in D) have extended
productivity. As awareness of these factors increases

along with the amount of time and labor spent in such 9


capacities, demand for talented interior designers and A) NO CHANGE
B) per year
ergonomic experts will grow.
C) every year
A good interior design is crucial for the aesthetic D) DELETE the underlined portion and end the
sentence with a period.
and expressive. At the same time, the ways in which we

use public and private spaces are changing quickly. Soon 10

we may need to work within our self-driving cars, or A) NO CHANGE


B) injuries and inefficiencies and distractions
turn spare bedrooms into home offices for C) injuries', and inefficiencies' and distractions'
telecommuting. As this becomes the rule rather than the D) injuries, and inefficiencies and distractions'

exception, and as traditional offices recognize the benefits


11
of human design, those with the interest, art, and skill to
Which choice best supports the statement made in
inspire with interior design will be sure to find success. the previous paragraph?
A) NO CHANGE
B) Some people think that the importance of
designs for employees' health is overrated.
C) Society has never been more aware of the
importance of design for health and happiness.
D) Companies are putting in greater investments
on developing better interior designs.

2
4

103
115
Q u
e
s
t
i
o
n
s
1
2
-
2
2
a
r
e
b
a
s
e
d
o
n
t
h
e
f
o
l
l
o
w i
n
g
p
a
s
s
a
g
e
.
12
A) NO CHANGE
Having Faith in Seismology
B) but
Imagine you are in Lisbon, Portugal, one of the C) also
D) but also
wealthiest regions in all of Europe. The date is November

1, 1755. The nation's economy is in full development,

and you are in one of the wealthiest regions in all of 13


Europe. Now imagine you are in Valdivia, Chile on A) NO CHANGE
May 22nd, 1960. It is a day much like any other day, B) that is
C) being
except for the beautiful spring weather. These seem like D) DELETE the underlined portion.
very different times and places, but they both share one

important factor: they will soon host two of the most


14
destructive earthquakes in recorded history, not only
Which choice most effectively combines the
changing the course of history for the nations they underlined sentences?
A) This fact is commonly known today but it
devastated but they also contributing to the
was controversial until as late as the 1950s,
development of the science of seismology. when it was proven through the use of fossil
records and sonar technology.
For much of our history, the cause of earthquakes B) While this fact is commonly known today, it
remained a mystery. The Lisbon earthquake, in particular, was controversial until as late as the 1950s
when it was proven through the use of fossil
was seen as a cruel punishment from God. Theodicy, records and sonar technology.
C) The use of fossil records and sonar technology
it is an entire branch of religious study, was even
proved the fact, however it was controversial
developed as a response to the apparent contradiction of until as late as the 1950s, and today this fact
is commonly known.
bad things happening to good people. We now know, D) It is commonly known today, but this fact was
however, that the movement and collision of Earth's controversial until it was proven through the
use of fossil records and sonar technology in
tectonic plates, not divine punishment, are the primary the late 1950s.
drivers of earthquakes. While this fact is commonly

known today, it was controversial until as late as the

1950s. It was proven through the use of fossil records

and sonar technology.

2
5

104
116
[1] Even if we now understand the mechanics behind 15

earthquakes, given there enormous size and devastation, A) NO CHANGE


B) its
what good will knowledge do? [2] This means that every C) it’s
increase in number represents an increase in energy D) their

10 times greater than the previous number. [3] Take, 16


for instance the moment magnitude scale, which is a A) NO CHANGE
revised version of the famous Richter scale. [4] The B) Take for instance,
3 C) Take, for instance,
Valdivia earthquake measured 9.5 on the moment D) For instance,

magnitude scale, a measurement so large that it was


17
recorded by a seismograph but not felt. [5] How can we
Which choice offers an accurate interpretation of the
possibly hope to prepare for something so unimaginably
data in figure 1?
A) NO CHANGE
large?
B) recorded a higher number than 1531 Lisbon
Fi
g
u
r
e
1 earthquake
C) measured 8.5
D) recorded a substantially smaller number than
1964 Rosais earthquake

18
Which choice offers an accurate interpretation of the
data in figure 2?
A) NO CHANGE
B) the event destroyed communities near the
epicenter.
C) the event caused a lot of damage in populated
F
igu
r
e
2 areas.
D) the event was felt, but only caused minor
damage.

19
To make this paragraph most logical, sentence 3
should be placed
A) where it is now.
B) after sentence 1.
C) after sentence 4.
D) after sentence 5.

2
6

105
117
In fact, it might be more than you would expect. 20
Learning about frequency, types, and effects of A) NO CHANGE
earthquakes allows builders to prepare. Even after the B) standards
C) examples
Lisbon earthquake all the way back in the 18th century, 3 D) principles
city planners began studying these effects. They constructed

small architectural models and simulated earthquakes by

marching troops through the room. The models that 21


survived served as the levels to rebuild the city. A) NO CHANGE
B) who
Combining good architecture with awareness of geographic C) they
risks in an area can greatly reduce the potential damage D) whom

of a major earthquake.

Unfortunately, many of these situations are seen as


22
abstract by the people which live there, and proper
The writer wants a conclusion that conveys how the
protective measures are expensive. For example, in the dangers of earthquake presented in the passage
should guide the actions of city planners. Which
Pacific Northwest, research indicates that there is a
choice best accomplishes this goal?
historically high risk of major earthquakes, but Seattle A) NO CHANGE
B) It must be acknowledged that earthquakes,
and Portland have yet to institute safety plans. despite the damages they have caused, have
enabled the construction industry to build
City planners are overly worried about the potential
houses that were impossible before.
dangers that earthquakes may cause. C) Many of the developments in city planning in
response to earthquakes have required trade-offs
in increased costs in building materials and
human labor.
D) City planners need to be made more aware of
these dangers or else face the kind of
consequences that people in Lisbon thought of
as "the wrath of God."

2
7

106
118
Questions 23-33 are based on the following passage 23
and supplementary material.
A) NO CHANGE
B) —computers and the Internet—
Disruptions of the New Technological Revolution C) —computers and the Internet
D) —computers and the Internet;
There have been at least three Industrial Revolutions (IR)

in the last 250 years. The first was driven by steam

power during the 19th century; the second was driven

by electricity during the turn of the century through the 24


World Wars; the third is more tentative, but can largely A) NO CHANGE
B) does,
be linked to digital information technology C) will,
—computers and the Internet, from the late 1960s D) did,

through the turn of the millennium. Increasing numbers

of historians argue that smartphone technology, automation,

and artificial intelligences represent a fourth IR, one

which may affect our society more profoundly than


3
the other ones do, so it is important to understand

what to expect.

2
8

107
119
Dr. Jim Parcel, a professor of history at Eastern 25
Michigan University. He says that "No matter how A) NO CHANGE
B) University; he says
one defines or views an Industrial Revolution: the C) University: he says
reason they are called 'revolutions' is because of the D) University, says

massive social changes that occur." "The lives

and livelihoods of enormous percentages of the population

are disrupted." Each of these shifts bring its own

benefits and drawbacks, and understanding them is 26


important, because the lessons we learn may make our A) NO CHANGE
B) Revolution, and
current transition easier. C) Revolution,
D) Revolution;
3

27
A) NO CHANGE
B) awesome
C) abnormal
D) mighty

28
A) NO CHANGE
B) bring their
C) brings their
D) brings its

2
9

108
120
The very first technological revolution resulted from 29
agriculture 12,000 years ago and remained the dominant The writer is considering deleting the previous
way of life among most human beings. The cultures, sentence. Should the writer make this change?
A) Yes, because it does not logically follow from
traditions, and social structures that define our lives and the previous paragraph.
B) Yes, because it introduces information that is
languages to this day were established during this time,
irrelevant at this point in the passage.
and were based around the needs and consequences of C) No, because it provides a logical introduction
3 to the paragraph.
farming. However, with the first IR, which began slowly D) No, because it provides an example in support
around 1760 and developed until around the 1840s, of statements made elsewhere in the passage.

society began to change.

The second IR of the late 19th century and early

20th century expanded all of this to an enormous degree,


30
and responded to the abuses of the first. Imbalances
Which choice most effectively supports the claim
between the rich and poor that had been unrestricted were made in the previous sentence?
A) Families moved from the country to cities,
fought against and regulated, and governments
organization happened around the workplace,
promised to more fairly and justly distribute the and people earned wages instead of goods.
B) The changes were considered significant at
gains made. that time, though it may not seem like much
by today's standards.
C) The changes with the society coincided with
the increases in agricultural productivities and
developments in technologies.
D) While the effects of those changes were felt
throughout the world, some people tried to
resist them.

31
A) NO CHANGE
B) promised to more fairly distribute the gains
made.
C) promised to distribute the gains made more
fairly and justly.
D) promised to distribute the gains more fairly,
even justly.

30

109
121
The third IR is tied to the rise of computers and the 32
internet. Communication, entertainment, and the nature of A) NO CHANGE
B) himself or herself
work itself began to change again. It decreased C) themselves
the need for the skilled labor of the second IR, and D) oneself

inequality began to rise again, with a great number leaving

the workforce altogether. Whether modern developments

like alternative energy sources and social media represent

a fourth IR or merely are an extension of the third is a

matter of debate, but Dr. Parcel says it may be something

even more compelling. "The rate of industrial development


33
has increased exponentially each time a breakthrough
The writer is considering revising the underlined
occurs," he said. "We may be advancing so rapidly we
portion of the sentence to read:
can no longer think in terms of 'revolution', and should
Automation in the form of robotics and
instead come to expect disruption within our own efficiency algorithms

lifetimes on a regular basis." Should the writer add this information here?
A) Yes, because it serves as a transitional point
in the paragraph.
B) Yes, because it provides examples of the
advancements that occurred during the third IR.
C) No, because it should be placed earlier in
the passage.
D) No, because it is not relevant to the main
claim of the passage.

31

110
122
Q u
e
s
t
i
o
n
s
3
4-4
4ar
e
b
a
s
e
d
o
n
t
h
e
f
o
l
l
o
w i
n
g
p
a
s
s
a
g
e
.
34
A) NO CHANGE
The Practice of Insulting the Meat B) meat: two
C) meat, two
Among the !Kung San hunter-gatherers of D) meat; two

southwest Africa, when a hunter returns to the tribe

with meat. Two things happen. First, the meat is

distributed among the entire tribe, especially to those 35


who need it the most. Second, the entire tribe begins to A) NO CHANGE
insult the meat for its quality, even those who are B) are
C) will
thankful for the charity. The more meat that is brought, D) has been
the longer and harsher the insults to the meat. In this

way, members of the tribe who would otherwise gain a


36
great deal of power due to their skills and strength
A) NO CHANGE
is, as the !Kung San say, "cooled." B) If the hunters are the neediest members of the
To many Western eyes, this practice may seem tribe,
C) If the hunters contribute to the success of a
bizarre, possibly even offensive. If they are to live group,
D) DELETE the underlined portion.
in the same community, they should be celebrated and

elevated above the others. This is an assertion associated

with cultures organized around individualistic principles,

such as the US or Europe. However, to collectivist


37
A) NO CHANGE
cultures, such as the !Kung San or many cultures in
B) the hunters seem ridiculous and are
East and Southeast Asia, the need for harmony underestimated.
C) it is more important to satisfy individual
outweighs the need for individual satisfaction. satisfaction than group harmony.
D) a person who contributes to the most to society
is seen as the most powerful.

32

111
123
Dr. Erin Stewart, director of the UNESCO 38
cultural exchange program said that "though each culture A) NO CHANGE
B) Stewart director of the UNESCO cultural
is unique, some generalities can be useful in trying to
exchange program said
understand why people from certain cultures behave the C) Stewart director of the UNESCO cultural
exchange program, said
way they do in certain situations." Stewart hopes that D) Stewart, director of the UNESCO cultural
by exploring these differences, greater cooperation can exchange program, said

be achieved between these cultures in business and 39


politics. It is best understood by studying The writer wants to link this paragraph with the
ideas that follow. Which choice best accomplishes
!Kung San culture. this goal?
[1] The idea is simple enough. [2] Each month, a A) NO CHANGE
B) She argues that we may be heading for a
single employee is selected for having performed above future in which we have a hybrid values
3
expectation relative to the peers, and his or her photo system
C) One example she uses to explain is the idea of
being placed on the wall. [3] Within an individualist the Employee of the Month.
D) Both systems have different advantages and
culture, this is seen as natural and appropriate, because the disadvantages in different situations.
culture values rewarding greater work with greater reward.
40
[4] Within a collectivist culture, however, the work
A) NO CHANGE
is seen as a group effort, and to single out an individual B) is
C) was
can be a source of shame or be seen as arrogant.
D) have been
[5] Without this information, a collectivist manager at
41
an individualist company might wonder why employees
A) NO CHANGE
were so frustrated with the removal of an incentive B) were so angry when the incentive program was
taken away from them,
program, while in the inverse situation, the individualist
C) became harsh critics of the removal of an
manager might be confused by the resentment suddenly incentive program,
D) lost their cool when an incentive program was
stirring within his collectivist employees. deleted,

42
To make this paragraph most logical, sentence 5
should be placed

A) where it is now.
B) before sentence 1.
C) before sentence 2.
D) before sentence 3.

33

112
124
While an individual may have different perspectives 43
on the relative merits of each value system, Dr. Stewart A) NO CHANGE
B) cautions
counsels against passing judgment. "Neither system C) admonishes
D) reprimands
is better or worse, and each has different advantages

and disadvantages in different situations." In fact, a

recent research study by the University of Texas shows 44


that harmony, efficiency, and quality of life are improved At this point, the writer is considering adding the
following sentence.
in both collectivist and individualist communities when
Greece, India, and Mexico exhibit collectivist
they are made aware of the existence of other value
cultures, while the US, Canada, and Germany
systems. "We may be heading for a future in which exhibit individualistic cultures.

we have hybrid value systems," she added. "Then we Should the writer make this addition here?

could literally have the best of both worlds." A) Yes, because it shows the link between
different value systems and different cultures.
B) Yes, because it lists countries associated with
those two opposite cultures.
C) No, because it provides an example that is
irrelevant to the paragraph.
D) No, because it fails to indicate why those
countries developed different value systems.

42

43

STOP
If you finish before time is called, you may check your work on this section only.

Do not turn to any other section.

34

113
125

You might also like